Download as pdf or txt
Download as pdf or txt
You are on page 1of 71

‫ ص‬12:33 2023/‫‏‬5/‫‏‬13 ASQ CMQ-OE : Practice Test # 09 ‫ ص‬12:33 2023/‫‏‬5/‫‏‬13 ASQ CMQ-OE : Practice Test # 09

1/1* Project status reports should be issued to various levels of stakeholders


ASQ CMQ-OE : Practice Test # 09 and on different aspects of project plans. Which of the following reasons
119/157 ‫إجمالي النقاط‬
behind issuing status reports is most important to achieving project
success?
Test yourself

To differentiate which team members are contributing to project success

To document progress to justify rewarding team members


* ‫عنوان بريد إلكتروني‬

sayed.rehan06@gmail.com To justify project expenditures

To monitor and adjust resources and priorities as needed

‫عنوان بال عنوان‬


‫التعليقات‬

Status reports let management and other stakeholders know if the project is on- or off-
track and how the project is being controlled to meet budget, scope, and schedule

.agreements

https://docs.google.com/forms/d/e/1FAIpQLSd9z1GvRQYUImApP8VPctKUexYoQbfBNUSA1dJW6a8m_6nB7Q/viewscore?vc=0&c=0&w=1&flr=0&view… 1/141 https://docs.google.com/forms/d/e/1FAIpQLSd9z1GvRQYUImApP8VPctKUexYoQbfBNUSA1dJW6a8m_6nB7Q/viewscore?vc=0&c=0&w=1&flr=0&view… 2/141


‫ ص‬12:33 2023/‫‏‬5/‫‏‬13 ASQ CMQ-OE : Practice Test # 09 ‫ ص‬12:33 2023/‫‏‬5/‫‏‬13 ASQ CMQ-OE : Practice Test # 09

1/1* A semi-conductor firm has experienced delays with purchased part lead 1/1* You are a project manager at a mid-sized hospital and have been asked
times, often making up a high proportion of overall lead times. The firm to develop draft process goals for the emergency department, which has
has provided feedback to the supplier, resulting in some improvements been the top source of patient complaints. What criteria would you use?
one month, and then delays the next month. The challenge for the semi-
conductor firm is to:
Goals should be SMART

Goals should be focused on improvement


Request ISO Certification to improve supplier performance.

Goals should focus on the processes of the emergency department


React to customer demands for the product during shipping delays.

Goals should reflect the desired end result


Spot negative trends in advance of a major problem.

Pursue lean and Just-in-Time (JIT) deliveries.

‫التعليقات‬

Process goals are aims,intents, targets or ends, supported by measurable objectives, and
‫التعليقات‬
.linked to the hospital's strategic plan. The identified issue has to do with end results

It is necessary to understand a supplier's operational performance metrics, so measuring


specific aspects of a supplier's processes can reveal potential financial, quality, and

.delivery issues

0/1* Staff members at a project-oriented business are asked to provide


management information each time an issue affecting the organization
hits the newswire. How could the negative effects of these requests be
mitigated?

Consider how information is communicated within the organization

Study how decisions are made within the organization

Create a template for the staff regarding collecting information

Have managers submit reports on the issues so staff can continue to work on
their jobs

‫اإلجابة الصحيحة‬

Study how decisions are made within the organization

https://docs.google.com/forms/d/e/1FAIpQLSd9z1GvRQYUImApP8VPctKUexYoQbfBNUSA1dJW6a8m_6nB7Q/viewscore?vc=0&c=0&w=1&flr=0&view… 3/141 https://docs.google.com/forms/d/e/1FAIpQLSd9z1GvRQYUImApP8VPctKUexYoQbfBNUSA1dJW6a8m_6nB7Q/viewscore?vc=0&c=0&w=1&flr=0&view… 4/141


‫ ص‬12:33 2023/‫‏‬5/‫‏‬13 ASQ CMQ-OE : Practice Test # 09 ‫ ص‬12:33 2023/‫‏‬5/‫‏‬13 ASQ CMQ-OE : Practice Test # 09

* You are an experienced human relations consultant and have been 1/1* Given the following SWOT analysis for a restaurant, which of the
contracted by a national distribution company to improve employee following actions provides the most potential (i.e., leverage) for
engagement. One of your first tasks was to review the company's improvement?
commitments to engagement. You found on their website and intranet
statements like "Our employees are our most important asset," "We have an • Strength: Strong relationships with vendors who provide high-quality
open-door policy", and "We want to hear from you." In addition, there is a ingredients help prevent running out of menu items.
seldom-used, Web-based suggestion system and an email address where
• Weakness: High staff turnover increases the need for a lot of training
employees can contact the CEO. How would you advise the company?
and makes customer service questionable as new employees are being
trained.

• Opportunity: Continuing to improve customer service will increase


Build employee engagement into the performance evaluation of everyone in a
position of management or leadership the likelihood of repeat customers and eventually increase sales.

Survey employees to identify the major gaps to be closed and then facilitate • Threat: A slump in the local economy is reducing customers' disposable
tactical teams to develop ways of closing those gaps income. This could lead to fewer customers eating out.

Develop an employee engagement training program for everyone who manages


employees
Initiate a catering service targeted toward local automotive manufacturing
Find out why the suggestion system is underused and take corrective action companies during holiday seasons

‫اإلجابة الصحيحة‬ Increase the number of vendors

Survey employees to identify the major gaps to be closed and then facilitate tactical Maintain customer service
teams to develop ways of closing those gaps
Improve retention of high-quality employees

‫التعليقات‬

Given that high staff turnover is a known weakness that is costing the organization money,

.targeting efforts to retain quality employees will yield better results than the other options

https://docs.google.com/forms/d/e/1FAIpQLSd9z1GvRQYUImApP8VPctKUexYoQbfBNUSA1dJW6a8m_6nB7Q/viewscore?vc=0&c=0&w=1&flr=0&view… 5/141 https://docs.google.com/forms/d/e/1FAIpQLSd9z1GvRQYUImApP8VPctKUexYoQbfBNUSA1dJW6a8m_6nB7Q/viewscore?vc=0&c=0&w=1&flr=0&view… 6/141


‫ ص‬12:33 2023/‫‏‬5/‫‏‬13 ASQ CMQ-OE : Practice Test # 09 ‫ ص‬12:33 2023/‫‏‬5/‫‏‬13 ASQ CMQ-OE : Practice Test # 09

1/1* One of Company ABC's sales representatives is popular for the highest
1/1 * When is stratified sampling useful for acquiring data?
rate of account wins, as well as the highest commission earnings.
However, it is also known within the company that all the accounts
solicited by this sales representative seem to have complex customer- When the customer requires 100% inspection be performed
specific requirements and are prone to error. The Quality Department
noted that most of the complaints are coming from this sales When only a particular portion of a population is to be evaluated
representative's accounts.
When conclusions need to be drawn about an entire population
Which of the following actions should the Quality Department take?
When nonconformances randomly occur throughout a lot

Increase the preventive action initiatives to minimize external errors


‫التعليقات‬
Increase the appraisal initiatives for early detection/prevention of errors
Stratified random sampling is a technique to segment a population prior to drawing a
random sample from each stratum. The stratum is usually defined so that the lack of
Review the total quality cost data to identify patterns

.homogeneity in the layers can be adjusted for in the sample selection

Evaluate returns and warranty claims

‫التعليقات‬ 1/1* One of your suppliers has just purchased a competitor and they are now
the only supplier producing a part that you need. What is the greatest risk
Reviewing total quality cost data will show patterns (if any) that will show the full picture
of an account's behavior. This will help focus the team's resources/energy in prioritizing
to your organization?

.areas needing improvement

Lack of part availability

Lower quality of the part

Decreased cost of the part

Increased cost of the part

‫التعليقات‬

Since this situation creates a monopoly, the supplier is likely to increase the price of the

.part

https://docs.google.com/forms/d/e/1FAIpQLSd9z1GvRQYUImApP8VPctKUexYoQbfBNUSA1dJW6a8m_6nB7Q/viewscore?vc=0&c=0&w=1&flr=0&view… 7/141 https://docs.google.com/forms/d/e/1FAIpQLSd9z1GvRQYUImApP8VPctKUexYoQbfBNUSA1dJW6a8m_6nB7Q/viewscore?vc=0&c=0&w=1&flr=0&view… 8/141


‫ ص‬12:33 2023/‫‏‬5/‫‏‬13 ASQ CMQ-OE : Practice Test # 09 ‫ ص‬12:33 2023/‫‏‬5/‫‏‬13 ASQ CMQ-OE : Practice Test # 09

* A quality manager has created an inspection plan for verifying conformance 0/1* As a key organizational leader with prior experience championing teams,
of a complex aircraft component assembly. The inspection plan was broken you have been tasked with implementing a change management
into a precise sequence of sub-task activities to ensure a high degree of approach for the organization. The organization's core business is
efficiency and time productivity. What management theory is the quality providing call center services to companies in the U.S. and Canada. Last
manager using? year, the organization began marketing and sales efforts to acquire
clients in Europe and China, and three new call centers have opened as
a result with more to follow.
Classical Organizational Theory
Top management wants the change management approach to
Behavioral Management Theory
accomplish the following:

Human Relations Theory


1. Transition all call centers to an empowered, natural work team
structure.
Scientific Management Theory
2. Identify and develop change agents within the call center workforce
that will drive continual improvement. 3. Establish buy-in on performance
metrics that will drive center efficiency and a high-degree of caller
‫التعليقات‬ satisfaction. You are given the resources to hire an external consultant as
Frederick Taylor's Scientific Management Theory emphasized efficiency by engineering
a change agent for the initiative.
processes to be repetitive, timely, and motion- efficient so they lead to high productivity.

.This is the approach used to develop the quality plan in this scenario One of your early tasks is interviewing and choosing the consultant.
Which of the following most influences your selection of the outside
consultant?

A SWOT analysis of each consultant

Consultant understanding of the differences between existing and new centers


within the context of your change initiative

Consultant experience with a natural work team structure

Consultant fluency with multiple global languages

‫اإلجابة الصحيحة‬

Consultant understanding of the differences between existing and new centers


within the context of your change initiative

https://docs.google.com/forms/d/e/1FAIpQLSd9z1GvRQYUImApP8VPctKUexYoQbfBNUSA1dJW6a8m_6nB7Q/viewscore?vc=0&c=0&w=1&flr=0&view… 9/141 https://docs.google.com/forms/d/e/1FAIpQLSd9z1GvRQYUImApP8VPctKUexYoQbfBNUSA1dJW6a8m_6nB7Q/viewscore?vc=0&c=0&w=1&flr=0&vie… 10/141


‫ ص‬12:33 2023/‫‏‬5/‫‏‬13 ASQ CMQ-OE : Practice Test # 09 ‫ ص‬12:33 2023/‫‏‬5/‫‏‬13 ASQ CMQ-OE : Practice Test # 09

0/1* As a newly appointed plant manager with a mandate to improve training 1/1* You are Quality Director for a large company. Your product line requires
to deal with long- standing performance issues, what would you do first? the company to follow strict government regulations in terms of product
safety and reporting potential issues. You have identified instances where
your company didn't follow regulations on reporting injuries that occurred
Develop a detailed training plan, focusing on specific skills needed to improve
with your products. The Legal Department is adamant that you don't
performance
disclose-even to management-what you found out because the company
Examine performance issues to see whether training can be part of the solution is exposed to risk.

Conduct a benchmarking survey to quantify the performance gap that your plant What would you do?
needs to close

Clarify the performance shortcomings with your direct reports and outline a Review practices of other, similar companies and be guided by their experience
timeframe for improvement
Refer the issue to the company's Ethics Committee
‫اإلجابة الصحيحة‬
Refer the issue to the company's Risk Assessment Department
Examine performance issues to see whether training can be part of the solution
Review laws and regulations governing disclosure and let them guide you

‫التعليقات‬

There will likely be laws and/or regulations governing disclosure. Compliance with them is

.paramount

https://docs.google.com/forms/d/e/1FAIpQLSd9z1GvRQYUImApP8VPctKUexYoQbfBNUSA1dJW6a8m_6nB7Q/viewscore?vc=0&c=0&w=1&flr=0&vie… 11/141 https://docs.google.com/forms/d/e/1FAIpQLSd9z1GvRQYUImApP8VPctKUexYoQbfBNUSA1dJW6a8m_6nB7Q/viewscore?vc=0&c=0&w=1&flr=0&vie… 12/141


‫ ص‬12:33 2023/‫‏‬5/‫‏‬13 ASQ CMQ-OE : Practice Test # 09 ‫ ص‬12:33 2023/‫‏‬5/‫‏‬13 ASQ CMQ-OE : Practice Test # 09

1/1* The operations manager led the planning and launching of a new 0/1* A company has a good manufacturing quality program in place, but a
business line with huge revenue potential. Initial failures like late customer has experienced a product failure. A process analysis of the
deliveries, mishandling of products, and excessive overtime-initially manufacturing quality program highlighted discrepancies and changes
perceived as part of the learning curve-persisted up to the project's ninth that needed to be made to:
monthly cycle. The company deployed one of its most expensive efforts
to salvage the business by dedicating key staff ranging from directors to
middle managers to perform hands-on tasks to help meet deadlines and
monitor the floor to prevent errors. The Director of Quality was tasked The quality control checks within the manufacturing process.
with compiling data surrounding the events so management can
Provide objective evidence of performance.
understand what went wrong with the product.
The source of the nonconforming product in the incoming raw materials.
Which of the following elements would the Director of Quality select as
the primary cause of this situation? The process capability of the manufacturing process.

‫اإلجابة الصحيحة‬
Project planning did not include historical comparisons with past successful
projects Provide objective evidence of performance.

The strategic planning team did not verify company capability vs. customer
requirements

Lean and Six Sigma concepts were not introduced to the system
0/1* For an important customer order, a supervisor gives the most
A seasoned workforce should have been dedicated to the project experienced injection-molding machine operator product specifications
and molding labor estimates. The molding machine's first- article runs did
not meet the tolerance parameters and the process is shut down. The
supervisor discusses this problem with the operator and states: "Please
‫التعليقات‬ troubleshoot the problem and make any adjustments you deem
Introduction of new projects normally impacts daily activities and may create strain on the necessary to achieve the required specifications and meet the labor
workforce. A review of customer requirements versus company capability (including skill ?estimates". This supervisor is using which leadership style
set, manpower, technology, etc.) should be performed to ensure the success of the

.strategic planning associated with the initiative

Low task, low relationship

Low task, high relationship

High task, low relationship

High task, high relationship

‫اإلجابة الصحيحة‬

Low task, low relationship

https://docs.google.com/forms/d/e/1FAIpQLSd9z1GvRQYUImApP8VPctKUexYoQbfBNUSA1dJW6a8m_6nB7Q/viewscore?vc=0&c=0&w=1&flr=0&vie… 13/141 https://docs.google.com/forms/d/e/1FAIpQLSd9z1GvRQYUImApP8VPctKUexYoQbfBNUSA1dJW6a8m_6nB7Q/viewscore?vc=0&c=0&w=1&flr=0&vie… 14/141


‫ ص‬12:33 2023/‫‏‬5/‫‏‬13 ASQ CMQ-OE : Practice Test # 09 ‫ ص‬12:33 2023/‫‏‬5/‫‏‬13 ASQ CMQ-OE : Practice Test # 09

1/1* Of the following options, the most important reason a maintenance 0/1* As part of the annual business planning process, the Quality Manager
department needs to give prompt attention to a malfunctioning machine is submitted a revised Quality Improvement (QI) plan-with a budget-to be
that: launched within the upcoming year. To improve the effectiveness of the
total quality system, one Ql project that was approved is the purchase
and implementation of a Quality Improvement System (QIS) software
Downtime is a waste for operations and manufacturing goals must be met. package to replace the current system, which contains numerous
database areas and uncontrolled spreadsheets, reports, and other Ql-
The treatment of internal customers influences the process for the external related documents. In addition, the current system is hard to search,
customer. making it difficult for new improvement teams or auditors to retrieve
Returning a malfunctioning machine to operation is a key metric for the specific information.
maintenance team.
The Quality Manager agreed to be the project manager for the initiative.
The malfunctioning machine will create defects and the number of customer Her financial acumen and general management skills are essential for the
complaints will increase. project to succeed. It was estimated that the project would take 30 weeks
5 ‫خيار‬ from planning to project close.

Which of the following project management activities will be more critical


in the early stage of the project?
‫التعليقات‬
Failure Mode and Effects Analysis (FMEA)
Regardless of the impacts resulting from the malfunction (product defects, line shutdown,
employee safety issues, etc.), the employees responsible for production are depending on
the company's maintenance department so production can resume as soon as possible. SWOT analysis
An unresponsive maintenance department may lead to apathy amongst internal

.customers, which may impact the way they treat their customers Force Field Analysis (FFA) diagram

Business case

‫اإلجابة الصحيحة‬

Failure Mode and Effects Analysis (FMEA)

https://docs.google.com/forms/d/e/1FAIpQLSd9z1GvRQYUImApP8VPctKUexYoQbfBNUSA1dJW6a8m_6nB7Q/viewscore?vc=0&c=0&w=1&flr=0&vie… 15/141 https://docs.google.com/forms/d/e/1FAIpQLSd9z1GvRQYUImApP8VPctKUexYoQbfBNUSA1dJW6a8m_6nB7Q/viewscore?vc=0&c=0&w=1&flr=0&vie… 16/141


‫ ص‬12:33 2023/‫‏‬5/‫‏‬13 ASQ CMQ-OE : Practice Test # 09 ‫ ص‬12:33 2023/‫‏‬5/‫‏‬13 ASQ CMQ-OE : Practice Test # 09

0/1* You are considering reorganizing your staff to better manage core 1/1* A new warehouse manager, trying to impress management in
customer needs. Which of the following should you know before addressing the backlog inherited from the previous manager, mandated a
determining how best to organize resources to accommodate core and drastic change in the workers' schedule to include working on weekends
non-core customers? without exception. Non-compliance to the new schedule meant a
reprimand or termination. Under this environment, productivity went up,
but the error rate doubled. Concerned that the growing error rate may
Customers lost and costs incurred due to the current organizational structure
eventually impact profit margins, the VP of Operations starts to look into
Position descriptions and skills of employees involved in the change the matter.

Customer analysis to determine the services and resources utilized by each Which of the following would be an effective action?

Return on Investment (ROI) for implementing the reorganization


Involve workers in creating a rotating weekend schedule during this transition
‫اإلجابة الصحيحة‬
Show visible support for the warehouse manager's decision to reinforce the new
Customer analysis to determine the services and resources utilized by each schedule

Reiterate the consequences of any violation to workers

Do nothing and leave the situation "as is" because it will eventually resolve itself

‫التعليقات‬

Management behavior towards internal customers has significant impact on the business.
Involving workers, as appropriate, promotes buy-in and participation in the program. If
human aspects of business are ignored, it is difficult to instill a desire to willingly

.)participate in achieving the overall goal (in this case, lessening the backlog

https://docs.google.com/forms/d/e/1FAIpQLSd9z1GvRQYUImApP8VPctKUexYoQbfBNUSA1dJW6a8m_6nB7Q/viewscore?vc=0&c=0&w=1&flr=0&vie… 17/141 https://docs.google.com/forms/d/e/1FAIpQLSd9z1GvRQYUImApP8VPctKUexYoQbfBNUSA1dJW6a8m_6nB7Q/viewscore?vc=0&c=0&w=1&flr=0&vie… 18/141


‫ ص‬12:33 2023/‫‏‬5/‫‏‬13 ASQ CMQ-OE : Practice Test # 09 ‫ ص‬12:33 2023/‫‏‬5/‫‏‬13 ASQ CMQ-OE : Practice Test # 09

1/1* A medical device company recently introduced a new product line and 1/1* As part of their continuous improvement program, the IT Department
expanded its customer base to include international territory. With these created a team called IT Solutions to obtain input from the workforce
changes, the board members voiced a concern about the continuing regarding issues causing delays and customer concerns. Feedback came
effectiveness of the company's overall strategic plan. What action would from multiple users of different software support systems representing
the executive team take to address the board members' concern? different phases of operations.

Under a limited resource budget, which of the following tools should the
Restrict individual functional areas from formulating their own different objectives IT team select as the optimum tool to process feedback?
to support the vision

Allow the process owners to drive the evaluation process; they are the subject
Activity Network Diagram (AND)
experts

Interrelationship digraph
Ensure that the span of the strategic plan is compatible with the changes

Affinity diagram
Solicit input from all functional teams in the review process

Matrix diagram

‫التعليقات‬

Input from all functional teams gives a wider view of support to drive the strategic ‫التعليقات‬

.planning
An affinity diagram is beneficial for organizing a large group of items into smaller chunks
that are easier to understand. This tool will also help group issues into what appear to be
similar categories so IT resources can be prioritized by category rather than individual
.issues

https://docs.google.com/forms/d/e/1FAIpQLSd9z1GvRQYUImApP8VPctKUexYoQbfBNUSA1dJW6a8m_6nB7Q/viewscore?vc=0&c=0&w=1&flr=0&vie… 19/141 https://docs.google.com/forms/d/e/1FAIpQLSd9z1GvRQYUImApP8VPctKUexYoQbfBNUSA1dJW6a8m_6nB7Q/viewscore?vc=0&c=0&w=1&flr=0&vie… 20/141


‫ ص‬12:33 2023/‫‏‬5/‫‏‬13 ASQ CMQ-OE : Practice Test # 09 ‫ ص‬12:33 2023/‫‏‬5/‫‏‬13 ASQ CMQ-OE : Practice Test # 09

1/1* Shipments of product are being returned due to mold contamination. The 0/1* Green Chem (GC) is a chemical distribution company. After 10
shipping director researched previous shipments and identifies several consecutive years of increasing revenue and profits, GC unexpectedly
that were rejected for the same reason. Which of the following tools lost several of its most valuable accounts, resulting in operating losses.
should the shipping director use to illustrate this data? Follow-up interviews with lost customers revealed GC Account Managers
lacked the necessary experience and knowledge to support them in their
industries. Top management realizes that they need to improve core
Flowchart
customer retention and loyalty and help Account Managers become
Histogram experts in customer industries. Several key elements of GC's new
strategic plan include:
Pareto chart
1. Reorganizing the sales force to support eight business units,
Cause and effect diagram developing greater industry knowledge and Account Manager
specialization.

2. Using customer relationship management by implementing customer


‫التعليقات‬ segmentation.
A Pareto chart is used to show relative frequencies or impacts of different categories. It
can illustrate how a reason for a problem compares to other reasons and establish that is 3. Adapting the annual Voice of the Customer (VOC) survey to focus on

.the most common, and therefore should be addressed first core customers identified in the new customer segments.

4. Expanding VOC outputs to provide actionable information to all levels,


not just top management.

5. Implementing a manager as coach culture to better support internal


1/1* Dolly's Home Cleaning offers many services, including pet waste pickup customers.
around the home. Lately, the owner has seen a bottleneck in bringing on
new customers. What is the rate at which the entire system generates GC expanded their VOC process with an email survey 2 years before
money through sales of services? segmentation was deployed. Moving forward, the annual survey will focus
only on core customers. With 120 super core and 1080 deluxe core
customers in each segment, survey methods must be adapted to ensure
Inventory
statistically valid samples are obtained from these population customer
segments.

Throughput
Which of the following survey design elements will have the most impact
Operating expense in increasing survey response rates?

Profit
Make sure the survey takes no longer than 15 minutes to complete

Attempting an email census of each core customer group instead of a sample


‫التعليقات‬
Use phone survey follow-ups for customers who don't respond to email invitations
Throughput is the rate at which the entire system generates money through sales of

.serviced. The other answers are not rate or flow related Attempt up to three e-mails to each customer rather than the historic single
attempt

‫اإلجابة الصحيحة‬

https://docs.google.com/forms/d/e/1FAIpQLSd9z1GvRQYUImApP8VPctKUexYoQbfBNUSA1dJW6a8m_6nB7Q/viewscore?vc=0&c=0&w=1&flr=0&vie… 21/141 https://docs.google.com/forms/d/e/1FAIpQLSd9z1GvRQYUImApP8VPctKUexYoQbfBNUSA1dJW6a8m_6nB7Q/viewscore?vc=0&c=0&w=1&flr=0&vie… 22/141


‫ ص‬12:33 2023/‫‏‬5/‫‏‬13 ASQ CMQ-OE : Practice Test # 09 ‫ ص‬12:33 2023/‫‏‬5/‫‏‬13 ASQ CMQ-OE : Practice Test # 09

Use phone survey follow-ups for customers who don't respond to email invitations
1/1* A company wishes to begin a partnership program with its customers.
The most likely group of customers to begin this process with would be
customers:
* The monthly customer complaint trend shows consistent recurrence of a
failure specific to a customer base. A team assigned to resolve the issue has That most affect the company's future.
recommended and implemented an action plan; the problem seemed solved
until the failure recurred after 6 months and again at 9 months. No one is With the greatest number of complaints against the company.
sure why there is a periodic recurrence of failure. The quality director was
That are the largest.
tasked to review the information and report any recommendations that will
help address the situation. That have been customers for the shortest amount of time.

Which of the following should the quality director recommend to


address/prevent the recurrence?
‫التعليقات‬

Redesign control chart tolerance The best partnerships are with customers who have the biggest impact on the future of

.the business
Perform a SIPOC analysis

Reevaluate the RPN value of the FMEA

Implement advanced Statistical Process Control (SPC) monitoring

1/1* Tacos Everywhere wants to provide the best tacos in the entire
metropolitan area. To ensure that they provide uniform products, what
‫التعليقات‬ should they provide to their potential suppliers first?
Using the SIPOC method (Supplier-Input-Process- Output-Customer) will help break down
the variables directly impacting the process. Since the failure seems to have a cyclical
pattern (3rd month, 6th month, 9th month), there may be a variable in the cyclical pattern Specifications
affecting the output. SIPOC is a simple tool that will help drill down the variables' influence

.on the output
Nondisclosure agreement

Customer survey data

Contract

‫التعليقات‬

Specifications help the supplier determine whether they can meet customer requirements.

.Specifications should include quality requirements and acceptance criteria

https://docs.google.com/forms/d/e/1FAIpQLSd9z1GvRQYUImApP8VPctKUexYoQbfBNUSA1dJW6a8m_6nB7Q/viewscore?vc=0&c=0&w=1&flr=0&vie… 23/141 https://docs.google.com/forms/d/e/1FAIpQLSd9z1GvRQYUImApP8VPctKUexYoQbfBNUSA1dJW6a8m_6nB7Q/viewscore?vc=0&c=0&w=1&flr=0&vie… 24/141


‫ ص‬12:33 2023/‫‏‬5/‫‏‬13 ASQ CMQ-OE : Practice Test # 09 ‫ ص‬12:33 2023/‫‏‬5/‫‏‬13 ASQ CMQ-OE : Practice Test # 09

1/1* A toymaker wants to assemble a focus group to better understand the 1/1* How could you best ensure that everyone in your multi-site company
required features of a new toy it is planning to introduce for children up to receives the same information and understanding about new changes to
3 years of age. Which of the following is the best focus group health care benefits?
arrangement?

Give all managers a summary of the changes and ask them to communicate this
information to their staff
Retailers and distributors are invited as the focus group because of their
experience with the toy business Have the health insurance company communicate it to each site
Parents with children in the target age group talk about the features that the toy
Send a written summary to all employees, followed by meetings at each
must have if they were to buy it for their children
location- with management present-to review changes and conduct Q&A
Children are provided a variety of toys to see which toys they use and like the
Focus on how the changes support the company's mission, vision, and strategic
most
objectives
Government authorities dealing with consumer safety and federal and state
environmental protection are invited to the focus group to provide their input

‫التعليقات‬

This is an important matter and should include the opportunity for two- way dialog-with
‫التعليقات‬
.management present-with a grounding in consistent written information in advance

Putting the respondent children in a position to interact with several toys is the best
approach. This gives the toymaker a chance to see which toys are most popular, how

.children play with the toys, and which features are used

https://docs.google.com/forms/d/e/1FAIpQLSd9z1GvRQYUImApP8VPctKUexYoQbfBNUSA1dJW6a8m_6nB7Q/viewscore?vc=0&c=0&w=1&flr=0&vie… 25/141 https://docs.google.com/forms/d/e/1FAIpQLSd9z1GvRQYUImApP8VPctKUexYoQbfBNUSA1dJW6a8m_6nB7Q/viewscore?vc=0&c=0&w=1&flr=0&vie… 26/141


‫ ص‬12:33 2023/‫‏‬5/‫‏‬13 ASQ CMQ-OE : Practice Test # 09 ‫ ص‬12:33 2023/‫‏‬5/‫‏‬13 ASQ CMQ-OE : Practice Test # 09

1/1* Which of the following will best overcome organizational barriers in the 1/1* A company has undergone some recent major organizational changes.
deployment of a knowledge management system? Identify what stage an existing team is in when the members decide to
discuss their concerns and differences about the changes.

Making the IT function the process owner of the knowledge management system
Forming
Making knowledge management deployment part of the vision and strategic
planning of the organization Performing

Focusing on integrating the needs of various internal processes into the knowledge
Norming
management system

Tasking the Quality Department with establishing criteria for knowledge Storming
management usage metrics

‫التعليقات‬
‫التعليقات‬
The team is in the norming stage; they are willing to discuss the recent changes and use
their discussion to resolve concerns and emotions about them. By doing so, they can
Incorporating knowledge management into the company vision and planning process

.return to the performing stage and work in alignment with the new organization
ensures top management support and formally provides the necessary organizational

.direction, resources, and alignment with other company systems

https://docs.google.com/forms/d/e/1FAIpQLSd9z1GvRQYUImApP8VPctKUexYoQbfBNUSA1dJW6a8m_6nB7Q/viewscore?vc=0&c=0&w=1&flr=0&vie… 27/141 https://docs.google.com/forms/d/e/1FAIpQLSd9z1GvRQYUImApP8VPctKUexYoQbfBNUSA1dJW6a8m_6nB7Q/viewscore?vc=0&c=0&w=1&flr=0&vie… 28/141


‫ ص‬12:33 2023/‫‏‬5/‫‏‬13 ASQ CMQ-OE : Practice Test # 09 ‫ ص‬12:33 2023/‫‏‬5/‫‏‬13 ASQ CMQ-OE : Practice Test # 09

1/1* Which of the following quality principles best aligns with a social service 1/1* Product safety is a huge component of overall quality, particularly due to
agency's quality system objective to "... provide legal support to parents product liability laws and unending lawsuits. How does the ASQ Code of
of children on the autism spectrum"? Ethics encompass product safety?

Offer qualified and relevant legal resources to all parents


Principle B.1
Become top of mind for all parents of autistic children
Principle C.4
Offer competent and valued legal resources to parents of autistic children
Principle A.2
Be friendly, be fair, and be fast in parental services
Principle A.5

‫التعليقات‬
‫التعليقات‬
The quality of the organization's activities will rest with the quality of services offered by it
and the value perceived by those who receive it. This statement describes the overall
Principle B.1 deals with holding paramount the safety, health, and welfare of the public in
intentions of the organization related to the quality of services for this segment of parents

.the performance of their professional duties

.served

https://docs.google.com/forms/d/e/1FAIpQLSd9z1GvRQYUImApP8VPctKUexYoQbfBNUSA1dJW6a8m_6nB7Q/viewscore?vc=0&c=0&w=1&flr=0&vie… 29/141 https://docs.google.com/forms/d/e/1FAIpQLSd9z1GvRQYUImApP8VPctKUexYoQbfBNUSA1dJW6a8m_6nB7Q/viewscore?vc=0&c=0&w=1&flr=0&vie… 30/141


‫ ص‬12:33 2023/‫‏‬5/‫‏‬13 ASQ CMQ-OE : Practice Test # 09 ‫ ص‬12:33 2023/‫‏‬5/‫‏‬13 ASQ CMQ-OE : Practice Test # 09

0/1* You are the Director of Quality and have recently worked with a third- 1/1* A steel fabricator's value stream map indicated that out of a total lead
party auditing group to pursue ISO registration. Due to the prevailing time of 22 weeks, only 1 week was spent doing true value-added work.
culture in your organization and the hard work you invested in designing This steel fabricator found that a large part of the nonvalue-added lead
the quality management system, the organization did well on the audit. time was identified as "waiting for approval" during many stages of the
order fulfillment process.
One surprising non-conformance that was issued concerned how
documents within the quality manual were reviewed and approved. Since How can the lead time be reduced?
you lead the Quality Department-and because your supervisor (the COO)
delegated the quality manual and the quality management system to you-
Standardize the work procedures to eliminate the need for many of the
you signed off on the revisions you introduced. As a result, you now need approvals
to revise the process and the related procedure. That said, you are also
serving as the interim Director of Training, so you need to ensure that the Mail an order confirmation to the customer to process the order more quickly
members of the Quality Department know about and implement the
Increase the cost of the order to eliminate customers unwilling to pay for the item
changes.
Develop a visual management system that shows the wait times for approval for
In a conversation with your supervisor (the COO), she asks if you are the each stage
ideal trainer for the new training. Your post-course surveys from other
previous training sessions indicate that most trainees were quite satisfied
with your training ability.
‫التعليقات‬
Which of the following Kirkpatrick's levels of evaluation is represented by
Because one of the key principles of lean thinking is to minimize the time between the
your post-course surveys? receipt of a customer order and fulfillment of that order, it's crucial to look at the entire

.lead time

Reaction

Behavior

Learning

Results

‫اإلجابة الصحيحة‬

Reaction

https://docs.google.com/forms/d/e/1FAIpQLSd9z1GvRQYUImApP8VPctKUexYoQbfBNUSA1dJW6a8m_6nB7Q/viewscore?vc=0&c=0&w=1&flr=0&vie… 31/141 https://docs.google.com/forms/d/e/1FAIpQLSd9z1GvRQYUImApP8VPctKUexYoQbfBNUSA1dJW6a8m_6nB7Q/viewscore?vc=0&c=0&w=1&flr=0&vie… 32/141


‫ ص‬12:33 2023/‫‏‬5/‫‏‬13 ASQ CMQ-OE : Practice Test # 09 ‫ ص‬12:33 2023/‫‏‬5/‫‏‬13 ASQ CMQ-OE : Practice Test # 09

0/1* The Vice President of Operations drafted plans to change the operational 1/1* Which of the following is a nonverbal or cultural factor that would affect
strategy supporting the company goal of two million dollars in cost communications between an auditor and an auditee?
savings over the next 2 years. This plan has not been formalized or
presented to the CEO for approval. What action would the Vice President
Age of the auditor
of Operations choose to get this proposal reviewed?
Facial expressions

Formulate the tactical plans needed for the proposal


Primary language difference between auditor and auditee

Assess internal forces capability, such as workforce skill set


Location of the onsite meeting with the auditee

Assess compatibility of the internal and external forces with the overall plan

Focus on operational plans to ensure readiness


‫التعليقات‬
‫اإلجابة الصحيحة‬ Facial expressions, or a lack thereof, are a form of nonverbal communication and can be

.interpreted differently by people from different cultures
Assess compatibility of the internal and external forces with the overall plan

https://docs.google.com/forms/d/e/1FAIpQLSd9z1GvRQYUImApP8VPctKUexYoQbfBNUSA1dJW6a8m_6nB7Q/viewscore?vc=0&c=0&w=1&flr=0&vie… 33/141 https://docs.google.com/forms/d/e/1FAIpQLSd9z1GvRQYUImApP8VPctKUexYoQbfBNUSA1dJW6a8m_6nB7Q/viewscore?vc=0&c=0&w=1&flr=0&vie… 34/141


‫ ص‬12:33 2023/‫‏‬5/‫‏‬13 ASQ CMQ-OE : Practice Test # 09 ‫ ص‬12:33 2023/‫‏‬5/‫‏‬13 ASQ CMQ-OE : Practice Test # 09

1/1* You have just started as Quality Director at a manufacturing firm with 0/1* As the training manager in a mid-sized manufacturing company, what
multiple locations, with 1500 employees total. You have found that would help managers build skills in providing feedback to problem
procedural documentation is lacking and if there is any, it has not been employees?
updated in a long time. There are known issues with too much internal
rework and a lack of planning resources and efforts. Your short list has 25
Remote learning with assignments completed at set intervals
possible areas to fix. The company COO has provided only a general
direction to increase capacity. Not all managers see the same issues, and Role-playing activities
disagreement is common between departments and up and down the
management chain. You have a large budget to work with as a large Computer-based training
potential customer has been identified, if you can meet their stringent
cost and quality needs. The company does not use the usual quality Lecture and discussion
tools, and you were hired to help.

‫اإلجابة الصحيحة‬
To start, you will need to gain more detail of leadership and mid-manager
Role-playing activities
needs. You used a maturity model and found results to be: Activities-
Unmanaged; Metrics-Standardized; Ownership-Repeated;
Interconnections-Managed; Changes-Repeated. What tool makes the
most sense to move forward?
1/1* Different customers have different needs, but your biggest competitors

Statistical Process Control (SPC) only offer "one size fits all" solutions. Which of the following will best help
you increase your company's share of the market?
Pareto chart

Just-in-time production control Hire additional salespeople to increase head-to-head competition

Cost of Quality Form a partnership with one of the smaller competitors to add their market share to
yours

Reengineer your processes to remove cost and offer similar products at a lower
price
‫التعليقات‬
Offer competitive products catering to distinct market niches
Common direction for leaders comes from the language of money, and cost of quality will
translate issues into costs. With this tool, alignment may be achieved to gain focus on
.what is most needed

‫التعليقات‬

By offering competitive products catering to several distinct market niches, the company

.can grow their total share of the market at the expense of the competitor

https://docs.google.com/forms/d/e/1FAIpQLSd9z1GvRQYUImApP8VPctKUexYoQbfBNUSA1dJW6a8m_6nB7Q/viewscore?vc=0&c=0&w=1&flr=0&vie… 35/141 https://docs.google.com/forms/d/e/1FAIpQLSd9z1GvRQYUImApP8VPctKUexYoQbfBNUSA1dJW6a8m_6nB7Q/viewscore?vc=0&c=0&w=1&flr=0&vie… 36/141


‫ ص‬12:33 2023/‫‏‬5/‫‏‬13 ASQ CMQ-OE : Practice Test # 09 ‫ ص‬12:33 2023/‫‏‬5/‫‏‬13 ASQ CMQ-OE : Practice Test # 09

1/1* Numerous legal and regulatory issues influence an organization's 1/1 * A leader for a self-managed team would be best selected by:
strategic planning. Which one of the following situations illustrates this
type of influence?
Team experience

A supervisor
A procedure is updated to address corrective actions initiated by an audit
Human resources
An industry association recommends updating a production standard
Team members
National requirements are amended due to numerous recalls within your
industry

A company scraps product because it discovers a defective part


‫التعليقات‬

A collective team decision establishes their ownership and support of their new team

.leader, further building the self- managed nature of the team

‫التعليقات‬

A national requirement could be a legal change in a law that could influence an



.organization's strategic planning

https://docs.google.com/forms/d/e/1FAIpQLSd9z1GvRQYUImApP8VPctKUexYoQbfBNUSA1dJW6a8m_6nB7Q/viewscore?vc=0&c=0&w=1&flr=0&vie… 37/141 https://docs.google.com/forms/d/e/1FAIpQLSd9z1GvRQYUImApP8VPctKUexYoQbfBNUSA1dJW6a8m_6nB7Q/viewscore?vc=0&c=0&w=1&flr=0&vie… 38/141


‫ ص‬12:33 2023/‫‏‬5/‫‏‬13 ASQ CMQ-OE : Practice Test # 09 ‫ ص‬12:33 2023/‫‏‬5/‫‏‬13 ASQ CMQ-OE : Practice Test # 09

1/1* A delay in a product delivery caused a customer's plant to shut down. 0/1* Dr. Juran detailed three key elements to quality management that
Due to the severity of this situation, the Quality Director initiates an became known as Juran's Trilogy. Which of the three would be applicable
investigation into the root cause of the failure. What is the appropriate to conceptualizing a system of quality checks and verifications to be
ethical approach in determining the validity of the analysis of the root applied during day-to-day operations?
cause investigation?

Quality Improvement (QI)


Regardless of the specific department that caused the delay, make the entire
company accountable Quality Assurance (QA)

Issue corrective actions to all departments involved in the process stream that Quality Control (QC)
resulted in the product delay
Quality Planning
Task those involved in product delivery with investigating the cause of the delay
‫اإلجابة الصحيحة‬
Create an independent team to investigate the issue
Quality Planning

‫التعليقات‬

Article 5 of the ASQ Code of Ethics states that quality professionals will act as faithful
agents or trustees and avoid conflicts of interest and the appearance of conflicts of 0/1* The director of training is scheduling training for the upcoming quarter.
interest. Creating an independent team comprised of the involved areas would be the best She has two instructors available to teach a single offering of a course.

.way to eliminate conflicts of interest
Fortunately, she has previous quarterly metrics available for each
instructor that are based loosely on Kirkpatrick's four evaluative levels.
How can she use the data to drive her instructor-selection decision?

She should select the instructor whose training resulted in the highest percentage
of observed behavioral change

She should select the instructor who most successfully met the learning objectives

She should select the instructor whose training resulted in the highest number of
positive results

She should select the instructor with the highest scores in post-course surveys

‫اإلجابة الصحيحة‬

She should select the instructor whose training resulted in the highest number of
positive results

https://docs.google.com/forms/d/e/1FAIpQLSd9z1GvRQYUImApP8VPctKUexYoQbfBNUSA1dJW6a8m_6nB7Q/viewscore?vc=0&c=0&w=1&flr=0&vie… 39/141 https://docs.google.com/forms/d/e/1FAIpQLSd9z1GvRQYUImApP8VPctKUexYoQbfBNUSA1dJW6a8m_6nB7Q/viewscore?vc=0&c=0&w=1&flr=0&vie… 40/141


‫ ص‬12:33 2023/‫‏‬5/‫‏‬13 ASQ CMQ-OE : Practice Test # 09 ‫ ص‬12:33 2023/‫‏‬5/‫‏‬13 ASQ CMQ-OE : Practice Test # 09

1/1* An audit team is exhibiting problematic personality differences and 0/1* Of the following techniques, what is the most effective at generating a list
members are growing concerned with how much work lies ahead. Many of both good and bad customer experiences?
feel overwhelmed and some groupthink is occurring. What stage of team
development are these behaviors most frequently associated with?
Interrelationship digraph

Critical Incident Technique

Performing
Outlier analysis

Accommodating
Fault tree analysis

Norming
‫اإلجابة الصحيحة‬

Storming
Critical Incident Technique

‫التعليقات‬

The behaviors listed are typical of the storming stage of team development. All teams 1/1* The training department is preparing material for "Right to Know" training

.exhibit storming issues, which are usually resolved in the norming stage on hazardous materials. There are no hazardous materials in the facility,
but company policy requires the training for legal purposes. Which
Bloom's Taxonomy level should the training department focus on for this
training material?

Evaluate
0/1* A continuous improvement team was given the task of optimizing the
Create
interchangeability of parts for the company's multiple product lines. Which
analysis tool should the team consider as the most appropriate first step? Analyze

Understand
Perform a Design for Manufacturing (DFMA) analysis

Initiate a brainstorming session

‫التعليقات‬
Perform a Design for Six Sigma (DFSS) analysis
The Understand level reflects a low level of complexity and requires the learner to be able
Create a storyboard of the product lines to summarize or give examples. This level of training will provide the workforce the
necessary hazardous materials knowledge that will most likely satisfy the stated legal and

.safety purpose
‫اإلجابة الصحيحة‬

Perform a Design for Manufacturing (DFMA) analysis

https://docs.google.com/forms/d/e/1FAIpQLSd9z1GvRQYUImApP8VPctKUexYoQbfBNUSA1dJW6a8m_6nB7Q/viewscore?vc=0&c=0&w=1&flr=0&vie… 41/141 https://docs.google.com/forms/d/e/1FAIpQLSd9z1GvRQYUImApP8VPctKUexYoQbfBNUSA1dJW6a8m_6nB7Q/viewscore?vc=0&c=0&w=1&flr=0&vie… 42/141


‫ ص‬12:33 2023/‫‏‬5/‫‏‬13 ASQ CMQ-OE : Practice Test # 09 ‫ ص‬12:33 2023/‫‏‬5/‫‏‬13 ASQ CMQ-OE : Practice Test # 09

1/1* Which of the following is a valid measurement that could be used to 0/1* Polly is a fully trained line lead at ABC Medical, Inc. She does her work
measure the success of the strategic objective "to increase efficiency"? well but is still asking her supervisor for advice and approval on decisions
more than necessary. The supervisor decides to address this by offering
more routine encouragement and coaching to build her confidence in her
Total number of employee suggestions implemented .own decision-making capabilities

Create one new product each year Which of the following situational leadership styles did the supervisor
apply?
Reduce computer downtime by 5%

Increase customer base by 100 new accounts


Low task, low relationship

High task, low relationship

‫التعليقات‬ Low task, high relationship

This metric encourages improvement by monitoring the number of employee suggestions


that get implemented. Presumably, the more suggestions implemented, the better the High task, high relationship

.organization will run, which in turn yields increased efficiency
‫اإلجابة الصحيحة‬

Low task, high relationship

https://docs.google.com/forms/d/e/1FAIpQLSd9z1GvRQYUImApP8VPctKUexYoQbfBNUSA1dJW6a8m_6nB7Q/viewscore?vc=0&c=0&w=1&flr=0&vie… 43/141 https://docs.google.com/forms/d/e/1FAIpQLSd9z1GvRQYUImApP8VPctKUexYoQbfBNUSA1dJW6a8m_6nB7Q/viewscore?vc=0&c=0&w=1&flr=0&vie… 44/141


‫ ص‬12:33 2023/‫‏‬5/‫‏‬13 ASQ CMQ-OE : Practice Test # 09 ‫ ص‬12:33 2023/‫‏‬5/‫‏‬13 ASQ CMQ-OE : Practice Test # 09

1/1* The warehouse receiving manager thinks that most employees want to 0/1* You are an experienced business analyst working for a mid-sized, family-
do their jobs well and will naturally look for problems as they arise in the owned manufacturer of custom-built store fixtures for high-end retailers.
warehouse and suggest solutions. Which of the following management The company has had a balanced scorecard in place for several years.
theories most appropriately describes this management style? Sales have not met expectations for the last four quarters, and profits are
down sharply. The owner has asked you to undertake a comprehensive
review of the results achieved. How should you start?
Behavioral Theory (Theory Y)

Learning Theory (Kolb) Determine whether each strategic goal and objective is supported by departmental
goals and objectives
Human Relations Theory (Hawthorne Effect)

Conduct customer surveys of the major accounts to determine their needs and
Classical Organizational Theory (Functional Structure)
expectations and why they are ordering less

Implement a company-wide Total Quality Management (TQM) program


‫التعليقات‬
Collect data from your company's trade association to help pinpoint reasons for
Behavioral Theory displaced the Theory X style that took a negative view of worker ability your performance decline
to self-regulate and assist in improvements. Theory Y takes a positive view of human
nature and believes workers want to do a good job and help change problems that affect ‫اإلجابة الصحيحة‬

.them personally
Determine whether each strategic goal and objective is supported by departmental
goals and objectives

https://docs.google.com/forms/d/e/1FAIpQLSd9z1GvRQYUImApP8VPctKUexYoQbfBNUSA1dJW6a8m_6nB7Q/viewscore?vc=0&c=0&w=1&flr=0&vie… 45/141 https://docs.google.com/forms/d/e/1FAIpQLSd9z1GvRQYUImApP8VPctKUexYoQbfBNUSA1dJW6a8m_6nB7Q/viewscore?vc=0&c=0&w=1&flr=0&vie… 46/141


‫ ص‬12:33 2023/‫‏‬5/‫‏‬13 ASQ CMQ-OE : Practice Test # 09 ‫ ص‬12:33 2023/‫‏‬5/‫‏‬13 ASQ CMQ-OE : Practice Test # 09

1/1* Customer RMY is using Company ABC as an extension of its facility 0/1* A customer provides a supplier with a monthly report containing
through the integration of its packaging and shipping operations within information on quality and delivery performance. This supplier provides a
Company ABC's facility. The raw materials are processed at the customer complicated electronic component. What additional information might be
site, then packaged and shipped from Company ABC. The operation has useful for the monthly report?
been stable for the last 3 years, but recently, defects are being reported
by end users. As partners, Customer RMY and Company ABC deployed
Defects
a cross-functional team to investigate.
Dun & Bradstreet or other available financial information
Which of the following would the team recommend to resolve this issue?
Technical support responsiveness

Tighten final inspection to prevent defects escape


ISO audit findings

Increase frequency of in-process audits until the issue is resolved


‫اإلجابة الصحيحة‬

Identify any change in the interrelated processes Technical support responsiveness

Review the company's receiving criteria

1/1* Which of the following team meeting situations most benefit from a
‫التعليقات‬
facilitator's intervention?
The cross-functional team should start by looking for changes in interrelated processes.
Doing so focuses on joint ownership of the problem rather than finger pointing or casting

.blame before a root cause is identified Not managing time allocated to the agendav

Inadequate capture of important data and discussions from meetings

Staffing and resource issues

Unstructured and off-topic discussions

‫التعليقات‬

The facilitator can help the team develop better discussion management techniques and
recommend quality tools to help add structure to their brainstorming and decision-making

.discussions

https://docs.google.com/forms/d/e/1FAIpQLSd9z1GvRQYUImApP8VPctKUexYoQbfBNUSA1dJW6a8m_6nB7Q/viewscore?vc=0&c=0&w=1&flr=0&vie… 47/141 https://docs.google.com/forms/d/e/1FAIpQLSd9z1GvRQYUImApP8VPctKUexYoQbfBNUSA1dJW6a8m_6nB7Q/viewscore?vc=0&c=0&w=1&flr=0&vie… 48/141


‫ ص‬12:33 2023/‫‏‬5/‫‏‬13 ASQ CMQ-OE : Practice Test # 09 ‫ ص‬12:33 2023/‫‏‬5/‫‏‬13 ASQ CMQ-OE : Practice Test # 09

1/1* A new customer focus team at Spicy Solutions, Inc. had its second 1/1* Management is reviewing projects for the upcoming year. During which
meeting where they began problem solving to determine the best way to level does management define and provide guidance on how a project is
satisfy their top customer segment. Jamie is the team's facilitator and she to be carried out?
is responsible for:

Conceptual
Running all team meetings.
Quantitative
Helping the team leverage diverse and multiple viewpoints.
Qualitative
Providing solution updates to the steering committee.
Operational
Developing and monitoring the project schedule.

‫التعليقات‬
‫التعليقات‬
The conceptual level is where objectives and metrics are used to define how actions are to
be done. This is the level that management defines actions to reach a successful project,
The facilitator ensures that everyone on the team can participate in idea generation and

.process, or product
that all ideas get heard. The facilitator may also recommend the use of quality tools like
affinity diagrams, mind-mapping, root cause analysis tools, or other data analysis tools for
.this purpose

1/1* In examining a delivery performance report from a key supplier, the


Quality Manager noticed that all orders from this supplier were delivered
late last month. The late deliveries of supplier products will most likely
result in which of the following?

Increased scrap and rework

Overtime

Late shipments to customers

The need to develop another supplier

‫التعليقات‬

Late supplier deliveries of key products or components will result in late final deliveries to

.customers even in a well- organized and well- planned process

https://docs.google.com/forms/d/e/1FAIpQLSd9z1GvRQYUImApP8VPctKUexYoQbfBNUSA1dJW6a8m_6nB7Q/viewscore?vc=0&c=0&w=1&flr=0&vie… 49/141 https://docs.google.com/forms/d/e/1FAIpQLSd9z1GvRQYUImApP8VPctKUexYoQbfBNUSA1dJW6a8m_6nB7Q/viewscore?vc=0&c=0&w=1&flr=0&vie… 50/141


‫ ص‬12:33 2023/‫‏‬5/‫‏‬13 ASQ CMQ-OE : Practice Test # 09 ‫ ص‬12:33 2023/‫‏‬5/‫‏‬13 ASQ CMQ-OE : Practice Test # 09

1/1* The Utah State Division of Elevator Inspection decided to achieve ISO 0/1* Customer service associates recently gave feedback to management.
certification. This will require standardizing best practices, writing Comments indicate that they would operate more efficiently if they had a
procedures, and training personnel to follow the procedures. The division more comprehensive understanding of the company's customer service
manager met with the three lead inspectors to determine what motivates guidelines and policies. They also stated they need more decision
each resource and uses that information to help the division accomplish authority as frontline customer representatives. How could the company
its goals. • Susan is working on her Project Management certification, so
?best address their concerns
the manager asks her to manage the overall project to achieve results by
.a specific deadline
Improve the customer service environment by adding a bulletin board that posts
• Fred is the most experienced and likes to engage with others when they the company's guidelines and policies
need assistance with performing technical inspections, so the manager
assigns him to a subject matter expert role. • Evelyn does not like to be Promote someone to a team leader to better coordinate and communicate with
the individual reps and management
micromanaged, so the manager defines her specific ISO tasks and
deadlines and allows Evelyn to self-determine how to complete her Set up a self-directed team that meets monthly to resolve these and other concerns
assignments by the deadline. that surface

The motivating factors for Susan, Fred, and Evelyn, respectively, are: Build better trust between customer service and management by initiating monthly
two-way meetings

‫اإلجابة الصحيحة‬
Achievement, power, and extrinsic needs.
Set up a self-directed team that meets monthly to resolve these and other concerns
Achievement, affiliation, and power.
that surface

Self-actualization, coaching, and power.

Achievement, self-actualization, and power.

‫التعليقات‬

Those who work to achieve personal goals are motivated by achievement, those who
engage with others are motivated by affiliation, and those who like control over their work
and authority are motivated by power. This scenario is a perfect example of the (David)
.McClelland Motivation Theory

https://docs.google.com/forms/d/e/1FAIpQLSd9z1GvRQYUImApP8VPctKUexYoQbfBNUSA1dJW6a8m_6nB7Q/viewscore?vc=0&c=0&w=1&flr=0&vie… 51/141 https://docs.google.com/forms/d/e/1FAIpQLSd9z1GvRQYUImApP8VPctKUexYoQbfBNUSA1dJW6a8m_6nB7Q/viewscore?vc=0&c=0&w=1&flr=0&vie… 52/141


‫ ص‬12:33 2023/‫‏‬5/‫‏‬13 ASQ CMQ-OE : Practice Test # 09 ‫ ص‬12:33 2023/‫‏‬5/‫‏‬13 ASQ CMQ-OE : Practice Test # 09

1/1 * Determine the appraisal costs in the following: 1/1* Which of the following is a critical area that should be addressed early in
developing a supplier certification contractual agreement between the
• $20,000 final test supplier and buyer?
• $350,000 warranty
Risk assessment
• $170,000 re-inspection
Specifications
• $45,000 inventory reduction
Classification of characteristics
• $4,000 incoming and source inspection
Inspection plans
• $30,000 rework

$200,000
‫التعليقات‬
$550,000
Risk assessment of the need to develop a long- term supplier, keep the work in-house, or
develop a short-term contract purchase should be the first consideration before engaging
$350,000
.in a supplier development process

$24,000

‫التعليقات‬


.Final testing ($20,000) and incoming and source inspection ($4,000) are appraisal costs

https://docs.google.com/forms/d/e/1FAIpQLSd9z1GvRQYUImApP8VPctKUexYoQbfBNUSA1dJW6a8m_6nB7Q/viewscore?vc=0&c=0&w=1&flr=0&vie… 53/141 https://docs.google.com/forms/d/e/1FAIpQLSd9z1GvRQYUImApP8VPctKUexYoQbfBNUSA1dJW6a8m_6nB7Q/viewscore?vc=0&c=0&w=1&flr=0&vie… 54/141


‫ ص‬12:33 2023/‫‏‬5/‫‏‬13 ASQ CMQ-OE : Practice Test # 09 ‫ ص‬12:34 2023/‫‏‬5/‫‏‬13 ASQ CMQ-OE : Practice Test # 09

1/1* Due to product failures during the first 2 months of product launch, the 0/1* As part of the annual business planning process, the Quality Manager
senior industrial engineer for Company ABC was asked to evaluate the submitted a revised Quality Improvement (QI) plan-with a budget-to be
process and verify process compliance with the product specification. It launched within the upcoming year. To improve the effectiveness of the
was confirmed that the process parameters are within the tolerance limit total quality system, one Ql project that was approved is the purchase
of Company ABC's internal procedures. To determine why the products and implementation of a Quality Improvement System (QIS) software
are being rejected, which of the following should the Quality Director package to replace the current system, which contains numerous
choose as the immediate, cost-effective next step? database areas and uncontrolled spreadsheets, reports, and other QI-
related documents. In addition, the current system is hard to search,
making it difficult for new improvement teams or auditors to retrieve
Verify raw materials are within the acceptable levels
specific information.
Conduct a customer survey for each shipment
The Quality Manager agreed to be the project manager for the initiative.
Tighten final inspection to prevent defects escape Her financial acumen and general management skills are essential for the
project to succeed. It was estimated that the project would take 30 weeks
Verify alignment of customer requirements and process parameters from planning to project close.

The project starts with the planning stage. The Quality Manager has
proceeded to assemble the project team that will complete all necessary
‫التعليقات‬ plans for review and approval at a stage gate meeting 30 days from now.
Mere compliance to internal procedures does not guarantee an acceptable product if the

.procedures are not aligned with customer requirements In addition to planning skills, what other management skills will the
Quality Manager need to rely on to complete the planning stage?

Staffing, organizing, situational leadership

Reporting, motivating, lessons learned

Directing, empowering, validating

Motivating, controlling, situational leadership

‫اإلجابة الصحيحة‬

Staffing, organizing, situational leadership

https://docs.google.com/forms/d/e/1FAIpQLSd9z1GvRQYUImApP8VPctKUexYoQbfBNUSA1dJW6a8m_6nB7Q/viewscore?vc=0&c=0&w=1&flr=0&vie… 55/141 https://docs.google.com/forms/d/e/1FAIpQLSd9z1GvRQYUImApP8VPctKUexYoQbfBNUSA1dJW6a8m_6nB7Q/viewscore?vc=0&c=0&w=1&flr=0&vie… 56/141


‫ ص‬12:34 2023/‫‏‬5/‫‏‬13 ASQ CMQ-OE : Practice Test # 09 ‫ ص‬12:34 2023/‫‏‬5/‫‏‬13 ASQ CMQ-OE : Practice Test # 09

1/1* If the upper and lower specification limits are set at 60, the process 1/1* Company EZ tasked the newly promoted Operations Manager to develop
capability ration (C.) is 2.0, and the process capability index (Cpk) is 1.5, a strategic plan to win a customer bid by offering lower costs with a
a process will: guarantee on best quality. 10 months after project launch, Company EZ
has not met the net profit expected from the project due to excessive
internal failures and expediting costs resulting from the pressure of
Produce no defects because process variation is so much less than the
specification limits. customer dissatisfaction with on-time delivery. The Director of Quality
was tasked with summarizing the lessons learned from this scenario to
Produce less than 3.4 defects per million even with normal shifts in the process prevent recurrence.
average.
Which of the following failure modes would the Director of Quality choose
Produce zero defects per million regardless of changes in the process average. as the primary root cause?

Always be stable because the process specification limits are well beyond the
process distribution. Failure in assessing capability vs. VOC requirement

Failure to adequately assess various options for reliable outsourcing

‫التعليقات‬ Failure to consider environmental threats

Even though the process will shift at a Cpk of 1.5, when the spread of the process is Failure in performing a tactical-level assessment
quantified with a Cp of 2.0, normal shifts will still provide a defect rate of less than 3.4
.ppm

‫التعليقات‬

Assessment of capability versus VOC requirement is vital in the initial contract review of a

.project. Focus on bidding for a lower cost may have undermined this requirement

https://docs.google.com/forms/d/e/1FAIpQLSd9z1GvRQYUImApP8VPctKUexYoQbfBNUSA1dJW6a8m_6nB7Q/viewscore?vc=0&c=0&w=1&flr=0&vie… 57/141 https://docs.google.com/forms/d/e/1FAIpQLSd9z1GvRQYUImApP8VPctKUexYoQbfBNUSA1dJW6a8m_6nB7Q/viewscore?vc=0&c=0&w=1&flr=0&vie… 58/141


‫ ص‬12:34 2023/‫‏‬5/‫‏‬13 ASQ CMQ-OE : Practice Test # 09 ‫ ص‬12:34 2023/‫‏‬5/‫‏‬13 ASQ CMQ-OE : Practice Test # 09

1/1* Brie Smith, the Operations Director, won the lottery and announced an 1/1* The management team required all departments to submit departmental
early retirement after 15 years of employment. This put the company in goals to support the new strategic plan involving process streamlining of
an urgent mode for finding her replacement. She is recognized by a new business unit.

management and peers to be a valued organizational leader. Which of


the following qualities should the hiring team consider as the "best fit" for Which of the following initiatives would the Quality Assurance Department
her replacement as a valued organizational leader? endorse as the primary initiative?

Review the guiding principles supporting the new strategic plan


Knowledgeable on international and domestic business laws

Review alignment of the quality policy with top-level policy


Regulatory experience in multiple product and safety standards

Audit the company's compliance with the existing quality program


Excellent document control skill set and writing skills

Create quality metrics to measure improvement initiatives


Systems thinker and change agent

‫التعليقات‬
‫التعليقات‬
Reviewing the alignment of the quality policy with the top-level policy is the logical starting
These qualities are needed in an organizational leader who can influence and provide

.point to ensure quality program/ objectives are synchronized with the new plan
.positive support to the achievement of important operational goals

https://docs.google.com/forms/d/e/1FAIpQLSd9z1GvRQYUImApP8VPctKUexYoQbfBNUSA1dJW6a8m_6nB7Q/viewscore?vc=0&c=0&w=1&flr=0&vie… 59/141 https://docs.google.com/forms/d/e/1FAIpQLSd9z1GvRQYUImApP8VPctKUexYoQbfBNUSA1dJW6a8m_6nB7Q/viewscore?vc=0&c=0&w=1&flr=0&vie… 60/141


‫ ص‬12:34 2023/‫‏‬5/‫‏‬13 ASQ CMQ-OE : Practice Test # 09 ‫ ص‬12:34 2023/‫‏‬5/‫‏‬13 ASQ CMQ-OE : Practice Test # 09

1/1* Theory of Constraints (TOC) emphasizes starting with which of the 1/1* The product development team at a smartphone manufacturer decides to
following principles? use a more expensive asymmetrical connector because a symmetrical
connector could be installed backwards. The team is utilizing which of the
following techniques?
Identifying the bottleneck in the process, maximizing its use, and pacing
production through it

Optimizing each operation in the process in order to optimize overall production


Quality Function Deployment (QFD)
Forming teams to look at the way people work and figuring out ways to eliminate
wasteful practices Kepner-Tregoe

Identifying all the bottlenecks in the plant and eliminating them Design for Six Sigma (DFSS)

DMAIC

‫التعليقات‬

TOC identifies the bottleneck, makes sure it is fully exploited and subordinates all other ‫التعليقات‬

.operations to it
An asymmetrical connector cannot be installed incorrectly and is a method of error

.proofing, which is an element of DFSS

1/1* How would a training manager ensure that a company had people
capable of filling in for others?

By designing all training modules using adult learning principles

Through cross-training on both a formal and informal level

Through a job enrichment campaign

Through the implementation of a formal mentor program

‫التعليقات‬

Cross-training enables employees to understand and perform each other's jobs. In a


healthy organizational culture, there may be an organic pollination of skills that occurs

.amongst employees, but a formal program is optimal to meet cross-training needs

https://docs.google.com/forms/d/e/1FAIpQLSd9z1GvRQYUImApP8VPctKUexYoQbfBNUSA1dJW6a8m_6nB7Q/viewscore?vc=0&c=0&w=1&flr=0&vie… 61/141 https://docs.google.com/forms/d/e/1FAIpQLSd9z1GvRQYUImApP8VPctKUexYoQbfBNUSA1dJW6a8m_6nB7Q/viewscore?vc=0&c=0&w=1&flr=0&vie… 62/141


‫ ص‬12:34 2023/‫‏‬5/‫‏‬13 ASQ CMQ-OE : Practice Test # 09 ‫ ص‬12:34 2023/‫‏‬5/‫‏‬13 ASQ CMQ-OE : Practice Test # 09

0/1* Based on the following fiscal data from the previous year, what is the * A manufacturing company meets their quarterly projected sales and profit
Return on Investment (ROI) and the Return on Assets (ROA) for these forecasts by rolling out new products and services. Which stakeholders are
activities? most affected by this?

• Purchased eight additional delivery trucks at total cost of $360,000 (paid


in cash) Employees

• The business owner loaned the company $400,000 to certify, staff, and Customers
operate the quality system for the previous year
Society
• Net income from operating the new trucks during the previous year was
Suppliers
$3,600,000

The quality system recorded savings of $1,200,000 in business costs


during the previous year
‫التعليقات‬


.Employees have a vested interest in continuing their source of income
ROI is 30% and ROI is $10:1

ROI is $3:1 and ROA is $10:1

ROI is 33% and ROA is $9:1

ROI is $3:1 and ROA is $9:1

‫اإلجابة الصحيحة‬

ROI is $3:1 and ROA is $10:1

https://docs.google.com/forms/d/e/1FAIpQLSd9z1GvRQYUImApP8VPctKUexYoQbfBNUSA1dJW6a8m_6nB7Q/viewscore?vc=0&c=0&w=1&flr=0&vie… 63/141 https://docs.google.com/forms/d/e/1FAIpQLSd9z1GvRQYUImApP8VPctKUexYoQbfBNUSA1dJW6a8m_6nB7Q/viewscore?vc=0&c=0&w=1&flr=0&vie… 64/141


‫ ص‬12:34 2023/‫‏‬5/‫‏‬13 ASQ CMQ-OE : Practice Test # 09 ‫ ص‬12:34 2023/‫‏‬5/‫‏‬13 ASQ CMQ-OE : Practice Test # 09

1/1* A firm needs to staff a customer complaint call center with associates 1/1* You are an experienced training course designer at a community college
who are mentally suited to the high-stress job. Of the following, what is and are working with a new customer who is a leader in warehousing and
the best way to accomplish this? distribution. This customer has asked you to develop a series of 1-hour
modules on employee engagement, to be delivered by their own
supervisors. How would you start?
Provide on-the-job training in an atmosphere like the call center

Enable the candidates to self-select after providing a "realistic job preview"


Ensure that the supervisors understand the training objectives

Ensure that the job description is up to date and requires the necessary background
Develop case studies to show why employee engagement is beneficial
to ensure that applicants are qualified
Develop a detailed Return on Training Investment (ROTI) to show the
Select the ideal candidates through psychological testing
benefitsDevelop a detailed Return on Training Investment (ROTI) to show the
benefits

Develop a detailed lesson plan for each module


‫التعليقات‬

It is more important to hire employees who have the right attitude than it is to hire
employees who appear to have the right skills but who may not have the right attitude.
Psychological testing is the best way to hire associates with the right personality traits for ‫التعليقات‬

.such a high-stress job
The first step within the training process/life cycle includes ensuring that the training
objectives are clearly defined and understood. Placing this action at the beginning of the
process ensures that you have determined and defined actual need(s), that supervisors
.understand and approve the learning objective(s), and, as a result, support the training

https://docs.google.com/forms/d/e/1FAIpQLSd9z1GvRQYUImApP8VPctKUexYoQbfBNUSA1dJW6a8m_6nB7Q/viewscore?vc=0&c=0&w=1&flr=0&vie… 65/141 https://docs.google.com/forms/d/e/1FAIpQLSd9z1GvRQYUImApP8VPctKUexYoQbfBNUSA1dJW6a8m_6nB7Q/viewscore?vc=0&c=0&w=1&flr=0&vie… 66/141


‫ ص‬12:34 2023/‫‏‬5/‫‏‬13 ASQ CMQ-OE : Practice Test # 09 ‫ ص‬12:34 2023/‫‏‬5/‫‏‬13 ASQ CMQ-OE : Practice Test # 09

0/1* The company's Beta-Boom project failed due to resource shortages and 1/1* What term best describes measuring equipment's performance across a
operational quality challenges not anticipated during the project planning specified range?
stage. Large financial losses occurred, so the company is revising the
process for new project launches as a corrective action. Which of the
following is the more appropriate planning focus to improve to prevent Precision

recurrence of this project failure?


Linearity

Improve project return on asset estimation accuracy Bias

Implement Program Evaluation and Review Technique (PERT) Accuracy

Project more accurate contribution estimates to the financial balance sheet

Improve project critical path analysis ‫التعليقات‬

‫اإلجابة الصحيحة‬ Linearity measures how well a piece of equipment performs across a specified operating

.range, by not showing unequal values when the scale has equal values
Implement Program Evaluation and Review Technique (PERT)

0/1* A Black Belt uses sample data to test if two machines are producing
* Your company is receiving raw material from a new supplier. You want to parts with the same central tendency. He erroneously concludes the two
monitor the incoming quality for all types of nonconformities. Setting the machines are producing parts that are different. This is called:
sample size to be the same amount, the proper control to use is the:

Type I error
u chart
Type II error
np chart
Consumer's risk
c chart
Beta error
p chart
‫اإلجابة الصحيحة‬

Type I error
‫التعليقات‬


.The c chart is to be used to denote defects data, with a constant sample size

https://docs.google.com/forms/d/e/1FAIpQLSd9z1GvRQYUImApP8VPctKUexYoQbfBNUSA1dJW6a8m_6nB7Q/viewscore?vc=0&c=0&w=1&flr=0&vie… 67/141 https://docs.google.com/forms/d/e/1FAIpQLSd9z1GvRQYUImApP8VPctKUexYoQbfBNUSA1dJW6a8m_6nB7Q/viewscore?vc=0&c=0&w=1&flr=0&vie… 68/141


‫ ص‬12:34 2023/‫‏‬5/‫‏‬13 ASQ CMQ-OE : Practice Test # 09 ‫ ص‬12:34 2023/‫‏‬5/‫‏‬13 ASQ CMQ-OE : Practice Test # 09

1/1* Due to rapid business growth, the executive team decided to evaluate the 1/1* As the newly hired director of training and development at a multi-
existing executive managers' span of control, to optimize resources location manufacturing company, one of your first tasks is to review an
without additional hiring of additional executives. Which of the following assortment of training that is now in place. What would be your overriding
tactics would be considered an effective approach for extending the question to ask in conducting this review?
managerial span of control with minimum leadership risk during the
?growth phase
Is the training the same at each location to ensure consistency and compliance
with customer requirements?

Allow convenience of where managers are located or based


Is the training aligned with the company's strategic and business needs?

Enhance flexibility and cooperation of the direct reports


Is there a schedule showing which people need to be trained and on what topics?

Increase level of the direct report's management competency and resources


Is there a clear return on training investment?

Implement a customer-departmentalized structure

‫التعليقات‬

‫التعليقات‬
.Training is most beneficial when it clearly supports the business direction of the company

Increasing the competency and resources of the direct reports has minimum risk and
requires less executive oversight. Better trained and more experienced middle
management tends to promote self-direction, ownership and autonomy of decision-
.making

1/1* Which of the following tools best help a team narrow down and focus on
key brainstormed ideas?

Affinity diagram

Activity Network Diagram (AND)

Interrelationship digraph

Nominal Group Technique

‫التعليقات‬

The Nominal Group Technique is used to narrow down a list of ideas by using rounds of
voting, preceded by discussion and debate. With each vote, the list is narrowed until the

.vital few ideas remain

https://docs.google.com/forms/d/e/1FAIpQLSd9z1GvRQYUImApP8VPctKUexYoQbfBNUSA1dJW6a8m_6nB7Q/viewscore?vc=0&c=0&w=1&flr=0&vie… 69/141 https://docs.google.com/forms/d/e/1FAIpQLSd9z1GvRQYUImApP8VPctKUexYoQbfBNUSA1dJW6a8m_6nB7Q/viewscore?vc=0&c=0&w=1&flr=0&vie… 70/141


‫ ص‬12:34 2023/‫‏‬5/‫‏‬13 ASQ CMQ-OE : Practice Test # 09 ‫ ص‬12:34 2023/‫‏‬5/‫‏‬13 ASQ CMQ-OE : Practice Test # 09

1/1 * Which of the following would best document project progress? 1/1* What is the primary purpose of a feedback system within the context of
project management responsibilities?

Approved requests for changes to project deliverables Identify opportunities for future team members

Budget reports, proposed milestones and their target dates Monitor, detect, report, and act on deviations from project plans

Meeting minutes, including members attending and hours billed Identify personnel who are responsible for both good and bad performance
deviations from plan
Completed deliverables per plan and monthly project summary reports
Monitor communications for rising conflicts, misinterpretation of documents and
requirements, and stakeholder rumors concerning project progress

‫التعليقات‬

Status of deliverables completed per plan and project summary reports explain project ‫التعليقات‬

.status including work, budget, time used, and time remaining
The primary purpose of a feedback system during the execution of a project is to monitor

.project plans and outcomes to make timely adjustments as necessary

https://docs.google.com/forms/d/e/1FAIpQLSd9z1GvRQYUImApP8VPctKUexYoQbfBNUSA1dJW6a8m_6nB7Q/viewscore?vc=0&c=0&w=1&flr=0&vie… 71/141 https://docs.google.com/forms/d/e/1FAIpQLSd9z1GvRQYUImApP8VPctKUexYoQbfBNUSA1dJW6a8m_6nB7Q/viewscore?vc=0&c=0&w=1&flr=0&vie… 72/141


‫ ص‬12:34 2023/‫‏‬5/‫‏‬13 ASQ CMQ-OE : Practice Test # 09 ‫ ص‬12:34 2023/‫‏‬5/‫‏‬13 ASQ CMQ-OE : Practice Test # 09

1/1* As a key, global quality leader new to the organization, you have been
tasked with implementing a quality strategy for the business. The
organization's core business is a pharmaceutical supplier of powdered
goods to customers across multiple countries in Europe, Asia Pacific, and
the Americas. Past Quality Management System (QMS) performance has 1/1* A casting process diagram is shown.
caused a loss of confidence for key customers.

The solid arrow line is the flow of physical units of product and the dotted
Top management wants an updated and relevant QMS to recover arrow line is the flow of kanban.
customer confidence and enable on-time delivery of quality goods
meeting relevant country and product safety protocols. You are required Which kind of kanban is this?
to:
Production kanban
1. Develop, deploy, and manage a QMS strategy across the global
organization. Job order kanban

2. Design and implement an effective measurement system that will Withdrawal kanban
reflect the performance of the QMS and its strategy.
Express kanban
One of your first tactics will be establishing your core QMS team. Which
of the following will help you to best determine which personnel can help
you with your tasks? Those with:
‫التعليقات‬

Withdrawal kanban specifies the kind and quantity of product which subsequent process
should withdraw from the preceding process. Withdrawing an item to the casting process

.is a signal one-way, and when the supplier replaces it is a signal the other way
Fluency with global languages.

Experience with an empowered natural work team structure.

Ability to conduct a SWOT analysis of the current QMS.

Long-term experience with the company, its production processes and its
customers.

‫التعليقات‬

The most important aspect the team members must have is an understanding and
capability to handle different needs across the full business. This information will best

.guide the quality leader as to who to retain for this project

https://docs.google.com/forms/d/e/1FAIpQLSd9z1GvRQYUImApP8VPctKUexYoQbfBNUSA1dJW6a8m_6nB7Q/viewscore?vc=0&c=0&w=1&flr=0&vie… 73/141 https://docs.google.com/forms/d/e/1FAIpQLSd9z1GvRQYUImApP8VPctKUexYoQbfBNUSA1dJW6a8m_6nB7Q/viewscore?vc=0&c=0&w=1&flr=0&vie… 74/141


‫ ص‬12:34 2023/‫‏‬5/‫‏‬13 ASQ CMQ-OE : Practice Test # 09 ‫ ص‬12:34 2023/‫‏‬5/‫‏‬13 ASQ CMQ-OE : Practice Test # 09

1/1* To survive a downturn in the economy across multiple business 1/1* A company has just launched a new product line that has resulted in a
segments, a global construction company needs to apply a 3-year 20% increase in complaints and 20 different proposed modifications and
strategic plan to take appropriate cost-reduction steps to survive. This improvements. Without hiring additional resources, which of the following
plan includes re-engineering business processes such as marketing, should the company use to prioritize resources to work on both
sales, branding, preconstruction, and estimating in order to achieve the complaints and modifications?
desired economic results. Why is strategic planning important for this
organization?
Assign some staff members to work on complaints and others on modifications

Organizations that use global strategic planning for reward and recognition are able Use a Pareto chart and an interrelationship digraph
to support the results of the planning process
Use a fishbone diagram and a contradiction matrix (from TRIZ)
The construction company will be able to better utilize its global internal
resources and grow the business Use root cause analysis and quality function deployment

Organizations that use global strategic planning don't have to establish standards
for measuring managerial performance

Organizations that use global strategic planning are unable to react to volatile ‫التعليقات‬
economic changes First, use the Pareto principle to help the company focus on the most important problems.
For each problem, an interrelationship digraph then helps identify relevant cause and
effect relationships among the critical issues so needed outcomes are at the center of an

.effective solution

‫التعليقات‬

Effective global strategic planning is an organizational management activity that is used


to articulate what an organization strives to become and the means to achieve it and

.assess and adjust the organization's direction in response to a changing environment

https://docs.google.com/forms/d/e/1FAIpQLSd9z1GvRQYUImApP8VPctKUexYoQbfBNUSA1dJW6a8m_6nB7Q/viewscore?vc=0&c=0&w=1&flr=0&vie… 75/141 https://docs.google.com/forms/d/e/1FAIpQLSd9z1GvRQYUImApP8VPctKUexYoQbfBNUSA1dJW6a8m_6nB7Q/viewscore?vc=0&c=0&w=1&flr=0&vie… 76/141


‫ ص‬12:34 2023/‫‏‬5/‫‏‬13 ASQ CMQ-OE : Practice Test # 09 ‫ ص‬12:34 2023/‫‏‬5/‫‏‬13 ASQ CMQ-OE : Practice Test # 09

1/1* You have been managing suppliers in your company for years, but 1/1 * What best describes the use of Quality Function Deployment (QFD)?
recently the company has made a change in the types of products that
they produce to improve profit and competitiveness. Your current
A structured method to ensure customer requirements are communicated
suppliers do not carry the components that you need and, with the help of
throughout an organization
Engineering and Purchasing, you are responsible for selecting several
new suppliers. A structured method for identifying critical control points in the process for
measuring customer requirements
Engineering has a list of suppliers that they have been using to create
A structured method for identifying failure modes and determining appropriate
prototypes of new products. The engineers purchased the components
actions to mitigate risk
on their own and did not involve anyone from Purchasing because the
engineers feel that the purchasing process is too cumbersome and slow. A structured method for determining the measurement error for determining
The timeline to produce this new product before competitors start conformance to customer requirements

production of a similar product is very short.

 One of your early tasks will be identifying the components used by


‫التعليقات‬
Engineering and working with Purchasing to find potential suppliers.
Which of the following risks are you likely to face? Deployment (QFD) is a structured method to gather and communicate customer and
product requirements throughout an organization. QFD does this by using a house of

.quality to translate customer requirements (needs/ wants) into operational requirements

Recall of your product

An abundance of suppliers for the components

Audit of your processes

Component availability

‫التعليقات‬

Component availability is a risk as Engineering was sourcing the components outside of


your purchasing system and some components may not be available through your supply

.base

https://docs.google.com/forms/d/e/1FAIpQLSd9z1GvRQYUImApP8VPctKUexYoQbfBNUSA1dJW6a8m_6nB7Q/viewscore?vc=0&c=0&w=1&flr=0&vie… 77/141 https://docs.google.com/forms/d/e/1FAIpQLSd9z1GvRQYUImApP8VPctKUexYoQbfBNUSA1dJW6a8m_6nB7Q/viewscore?vc=0&c=0&w=1&flr=0&vie… 78/141


‫ ص‬12:34 2023/‫‏‬5/‫‏‬13 ASQ CMQ-OE : Practice Test # 09 ‫ ص‬12:34 2023/‫‏‬5/‫‏‬13 ASQ CMQ-OE : Practice Test # 09

1/1 * The major purpose of a benefit-cost analysis is to: 1/1* You have been managing suppliers in your company for years, but
recently the company has made a change in the types of products that
they produce to improve profit and competitiveness. Your current
Calculate the net project gain (or loss).
suppliers do not carry the components that you need and, with the help of
Identify project benefits.
Engineering and Purchasing, you are responsible for selecting several
new suppliers.
Estimate the cost of the project.
Engineering has a list of suppliers that they have been using to create
Determine if a project is financially viable. prototypes of new products. The engineers purchased the components
on their own and did not involve anyone from Purchasing because the
engineers feel that the purchasing process is too cumbersome and slow.
The timeline to produce this new product before competitors start
‫التعليقات‬
production of a similar product is very short.
This analysis is a comparison of dollar value of benefits derived from an initiative to
associated costs incurred to realize those benefits over some period. A common term Now that several new suppliers have been selected, how would you
used for this is Return on Investment or ROI. This type of analysis helps determine if

.projects are initiated or put on hold during portfolio management decision making
make sure they understand your business needs?

Onboarding

Track metrics over time

Provide feedback when issues occur

Host a special event

‫التعليقات‬

Onboarding is the process of assimilating the supplier to your business processes. This is

.important so suppliers become acclimated to your organization and key business needs

https://docs.google.com/forms/d/e/1FAIpQLSd9z1GvRQYUImApP8VPctKUexYoQbfBNUSA1dJW6a8m_6nB7Q/viewscore?vc=0&c=0&w=1&flr=0&vie… 79/141 https://docs.google.com/forms/d/e/1FAIpQLSd9z1GvRQYUImApP8VPctKUexYoQbfBNUSA1dJW6a8m_6nB7Q/viewscore?vc=0&c=0&w=1&flr=0&vie… 80/141


‫ ص‬12:34 2023/‫‏‬5/‫‏‬13 ASQ CMQ-OE : Practice Test # 09 ‫ ص‬12:34 2023/‫‏‬5/‫‏‬13 ASQ CMQ-OE : Practice Test # 09

1/1* An automobile manufacturer wants to gather data to aid its new strategy 0/1* The formalized team approach to getting things done replaces which of
to improve the support it provides to its most vital dealers, while also the following management concepts for dispersing project task
improving new car owner satisfaction, retention, and loyalty. The assignments?
manufacturer has 3,000 dealers nationwide, of which the top 50
accounted for over 70% of cars sold last year. Which of the following
Worker empowerment
approaches is likely to be most effective?
Total Quality Management (TQM)

Create a detailed survey and send it to all customers who purchased cars last year
Theory X

Use data from publications like Consumer Reports to be certain about customer
Hoshin planning
needs and wants and how to improve customer satisfaction

Send an email survey to all dealers nationally to gather feedback on opportunities ‫اإلجابة الصحيحة‬
to provide better service to dealers and car buyers
Theory X
Conduct phone surveys with all 50 core dealers to determine unmet needs and
their ideas to improve buyer satisfaction, retention and loyalty and follow up
with each new car purchaser to ensure initial quality exceeds expectations

0/1*  Which of the following leader type gains followers to pursue a goal
through leveraging motivation, communicating in the language of the
‫التعليقات‬ followers, and using the power of persuasion?
Using phone surveys will get a very high response from core dealers and data quality will
be exceptional. In addition, a customer email survey 10- 14 days after buyer purchase is a
good way to ensure that initial quality is high and it should help improve retention and Situational leader

.loyalty
Cause leader

Transformational leader

Transactional leader

‫اإلجابة الصحيحة‬

Cause leader

https://docs.google.com/forms/d/e/1FAIpQLSd9z1GvRQYUImApP8VPctKUexYoQbfBNUSA1dJW6a8m_6nB7Q/viewscore?vc=0&c=0&w=1&flr=0&vie… 81/141 https://docs.google.com/forms/d/e/1FAIpQLSd9z1GvRQYUImApP8VPctKUexYoQbfBNUSA1dJW6a8m_6nB7Q/viewscore?vc=0&c=0&w=1&flr=0&vie… 82/141


‫ ص‬12:34 2023/‫‏‬5/‫‏‬13 ASQ CMQ-OE : Practice Test # 09 ‫ ص‬12:34 2023/‫‏‬5/‫‏‬13 ASQ CMQ-OE : Practice Test # 09

* Of various types of strategic planning processes, which of the following 1/1* A machine manufacturer wants to ensure the quality and integrity of the
statements are best reflective of scenario planning weld in an assembly from its supplier. Which one of the following is the
best option to accomplish this objective?

A planning process based on formulation of the strategies and objectives along


with the execution of tactical and operational plans Require the supplier to provide the welding and inspection processes, which will
be utilized as another level of ensuring conformance to requirements
A planning process which involves every member of the organization and utilizes a
catchball methodology The machine manufacturer should develop the welding process

A planning process based on a disciplined and structured approach where good


Require that the supplier achieve ISO 9001 certification prior to contract award
and bad futures and wild cards are considered
Ensure that the supplier uses only certified welders and maintains records to prove
A planning process based of the conducting internal and external environmental
it
scans relevant to the business

‫التعليقات‬
‫التعليقات‬
There are various welding processes available and the integrity of the weld might be

.This statement has the three macro scenarios of scenario planning critical in the early welding stages, which can then be cosmetically improved after welding

.is complete

https://docs.google.com/forms/d/e/1FAIpQLSd9z1GvRQYUImApP8VPctKUexYoQbfBNUSA1dJW6a8m_6nB7Q/viewscore?vc=0&c=0&w=1&flr=0&vie… 83/141 https://docs.google.com/forms/d/e/1FAIpQLSd9z1GvRQYUImApP8VPctKUexYoQbfBNUSA1dJW6a8m_6nB7Q/viewscore?vc=0&c=0&w=1&flr=0&vie… 84/141


‫ ص‬12:34 2023/‫‏‬5/‫‏‬13 ASQ CMQ-OE : Practice Test # 09 ‫ ص‬12:34 2023/‫‏‬5/‫‏‬13 ASQ CMQ-OE : Practice Test # 09

0/1* Process performance has been declining and threatens the loss of a 1/1* You are assigned to manage a multi-skilled, high-performance training
critical account. Most of the failures are due to processing errors caused program. What would you review to measure the overall effectiveness of
by workforce non-compliance. As a last resort, a review of instructions for the training program?
clarity, increased floor supervision, and disciplinary actions have been
deployed to provide guidance and support to the workforce. These efforts
Customer satisfaction
have resulted in little to no change in performance.
Previous training assessment data
Which of the following quality initiatives should the Quality Director
choose as most appropriate to address this scenario? Instructor performance

Trainee satisfaction
Ensure that the quality policy is accessible and visible to the workforce for
reference

Inventory and update job descriptions to ensure quality objectives are included
‫التعليقات‬
Create an approval system for planned, temporary deviations so that workforce
By reviewing as much data as possible, you have a better probability of gathering a more
activities will not go out of control complete picture of program effectiveness. Reviewing front-facing elements (e.g., did
training drive customer satisfaction) with rear- facing elements (e.g., did trainees see
Validate the consistency and understanding of the quality objectives throughout the value within the training, was the delivery mode optimal, etc.), can improve the visibility
company
.gleaned through measurements

‫اإلجابة الصحيحة‬

Validate the consistency and understanding of the quality objectives throughout the
company

https://docs.google.com/forms/d/e/1FAIpQLSd9z1GvRQYUImApP8VPctKUexYoQbfBNUSA1dJW6a8m_6nB7Q/viewscore?vc=0&c=0&w=1&flr=0&vie… 85/141 https://docs.google.com/forms/d/e/1FAIpQLSd9z1GvRQYUImApP8VPctKUexYoQbfBNUSA1dJW6a8m_6nB7Q/viewscore?vc=0&c=0&w=1&flr=0&vie… 86/141


‫ ص‬12:34 2023/‫‏‬5/‫‏‬13 ASQ CMQ-OE : Practice Test # 09 ‫ ص‬12:34 2023/‫‏‬5/‫‏‬13 ASQ CMQ-OE : Practice Test # 09

1/1* The QA department is not meeting its goal of investigating complaints in 1/1* When reviewing milestone status, a construction project's costs have
a timely manner, which is delaying promised customer responses. Which exceeded budget as of that time. Half of the total work has been
of the following would best improve QA customer service? accomplished to date, which indicates that the project is behind schedule
and may go over budget. How should the project manager act to try to
keep the project on time and within budget at the end?
Increase the timeframe for when a response is promised to the customer based on
current performance

Analyze the investigational process to determine the cause for the delays and Immediately request authorization to increase the budget for more labor resources
identify improvements to streamline information flow
Do nothing because projects tend to self-correct as future tasks are managed to
Assign additional QA staff to investigate complaints until the goal is met and the save time or money
number of complaints has stabilized
Conduct an Earned Value Analysis (EVA) to determine the extent of the problem
Make sure timely response is added to individual performance goals so individuals and negotiate with stakeholders to determine best corrective action
are held accountable for meeting the goal
Request authorization to compress a large, upcoming task that is not on the critical
path

‫التعليقات‬

The first step should be to look at flows through the QA department and the reasons for ‫التعليقات‬
delays in investigating complaints. Once causes have been determined and improvements

.made, verify that customer promise dates are now being met Until the EVA state is better analyzed, the PM does not know how much work remains vs.
budget. When making necessary adjustments to a project, costs, schedule, deliverables,
and quality requirements must be addressed. By bringing the resolution process to
stakeholders that will benefit from the project, the PM will determine the best way to

.correct the current state, including any necessary adjustments to the project's charter

https://docs.google.com/forms/d/e/1FAIpQLSd9z1GvRQYUImApP8VPctKUexYoQbfBNUSA1dJW6a8m_6nB7Q/viewscore?vc=0&c=0&w=1&flr=0&vie… 87/141 https://docs.google.com/forms/d/e/1FAIpQLSd9z1GvRQYUImApP8VPctKUexYoQbfBNUSA1dJW6a8m_6nB7Q/viewscore?vc=0&c=0&w=1&flr=0&vie… 88/141


‫ ص‬12:34 2023/‫‏‬5/‫‏‬13 ASQ CMQ-OE : Practice Test # 09 ‫ ص‬12:34 2023/‫‏‬5/‫‏‬13 ASQ CMQ-OE : Practice Test # 09

0/1* The corrective action issued to one of Company ABC's suppliers seems 1/1* As a result of its long history of quality improvement initiatives, a hospital
to be ineffective as the defect keeps repeating. What next step should the received the Malcolm Baldrige National Quality Award. They established
customer take with the supplier to address this issue? a framework that focused on proactive benchmarking, a smaller and well
managed supplier base, a culture of patient focus, Baldrige category
ownership teams, and a more robust system of healthcare Quality
Send a technical representative to assist the supplier
Assurance (QA), Quality Control (QC), and Quality Improvement (QI)
best practices. This framework forms the basis for:
Provide continuous improvement training modules to the supplier

Review root cause analysis with supplier


Rapid cycle Plan-Do-Check-Act problem solving.
Audit the supplier's facility
Cost control management.
‫اإلجابة الصحيحة‬
A commitment to zero defects.
Review root cause analysis with supplier
Total quality management.

1/1* Changes in technology may impact an organization's short- and long- ‫التعليقات‬
term strategic plans by requiring the organization to:
TQM is general term for an integrated system of quality management practices, commonly
guided by a standard like the Baldrige criteria or ISO 9001. An integrated system connects
best practices of quality assurance, control, and improvement to exceed the expectations
Consider the effect on business processes.
.of customers, employees, and other stakeholders

Increase organizational security plans.

Increase advertising product services.

Limit financial auditing.

‫التعليقات‬

Before any changes are implemented to a system or business process, their potential
effects (positive and negative) should be assessed to ensure the changes do not

.negatively impact users

https://docs.google.com/forms/d/e/1FAIpQLSd9z1GvRQYUImApP8VPctKUexYoQbfBNUSA1dJW6a8m_6nB7Q/viewscore?vc=0&c=0&w=1&flr=0&vie… 89/141 https://docs.google.com/forms/d/e/1FAIpQLSd9z1GvRQYUImApP8VPctKUexYoQbfBNUSA1dJW6a8m_6nB7Q/viewscore?vc=0&c=0&w=1&flr=0&vie… 90/141


‫ ص‬12:34 2023/‫‏‬5/‫‏‬13 ASQ CMQ-OE : Practice Test # 09 ‫ ص‬12:34 2023/‫‏‬5/‫‏‬13 ASQ CMQ-OE : Practice Test # 09

1/1* Which of the following are the most representative samples of classical 1/1* Green Chem (GC) is a chemical distribution company. After 10
market forces? consecutive years of increasing revenue and profits, GC unexpectedly
lost several of its most valuable accounts, resulting in operating losses.
Follow-up interviews with lost customers revealed GC Account Managers
Buyer bargaining power, employee engagement and satisfaction, supplier lacked the necessary experience and knowledge to support them in their
consolidation
industries. Top management realizes that they need to improve core
Global market changes, organizational position for growth, supplier bargaining customer retention and loyalty and help Account Managers become
power experts in customer industries. Several key elements of GC's new
strategic plan include: 1. Reorganizing the sales force to support eight
Competitor rivalry, market seasonality, threat of substitutes and obsolescence
business units, developing greater industry knowledge and Account
Global market changes, competitor rivalry and buyer bargaining power Manager specialization.

2. Using customer relationship management by implementing customer


segmentation.
‫التعليقات‬
3. Adapting the annual Voice of the Customer (VOC) survey to focus on
These are three of the six types of classical market forces where the other three are threat core customers identified in the new customer segments.

.of substitutes/obsolescence, supplier bargaining power, and new competitor entry

4. Expanding VOC outputs to provide actionable information to all levels,


not just top management.

5. Implementing a manager as coach culture to better support internal


customers.

You and your teams have found many causes for the organization's
issues, and solutions for them as well. The best solutions have been
selected and agreed on by all levels of management. Rapid
implementation is underway, but you find that many areas keep going
back to their old practices. What tools or techniques are best to apply at
this stage?

Retraining, worker penalties, and motivational posters in the break room

Flowcharting processes, updating procedures, and creating new work


instructions

Incentive plans, training, and coaching on undesirable behaviors

Time studies, reporting of errors by coworkers, and suggestion boxes

‫التعليقات‬

In the scenario, it was stated that "procedure documentation has been spotty and not
updated." This past practice would show that better work instructions would be required,
and the "rapid implementation" hints that procedures and work instructions may not have
https://docs.google.com/forms/d/e/1FAIpQLSd9z1GvRQYUImApP8VPctKUexYoQbfBNUSA1dJW6a8m_6nB7Q/viewscore?vc=0&c=0&w=1&flr=0&vie… 91/141 https://docs.google.com/forms/d/e/1FAIpQLSd9z1GvRQYUImApP8VPctKUexYoQbfBNUSA1dJW6a8m_6nB7Q/viewscore?vc=0&c=0&w=1&flr=0&vie… 92/141
‫ ص‬12:34 2023/‫‏‬5/‫‏‬13 ASQ CMQ-OE : Practice Test # 09 ‫ ص‬12:34 2023/‫‏‬5/‫‏‬13 ASQ CMQ-OE : Practice Test # 09

.been updated, or not updated well

1/1* The development of a customer-supplier partnership can be a very


involved process that requires a significant amount of time to develop.
What are the critical activities that signify that a customer-supplier
partnership is already in place?

0/1* Appropriate communication methods related to project plans and The customer invites the supplier to become involved in the early stages of a
activities should: customer's new product development process

The supplier selection process ends, and an initial contract is given that includes
Be written from the perspective of the project's vertical authority to ensure that requirements mandated by an agreed-upon quality plan
O information used by lower-level colleagues matches the information used by
The supplier begins to receive longer-term contracts for larger volumes of products
higher-level leaders.
and services on a regular basis
Only be distributed to the parties responsible for doing the activities and making
A long-term contract is executed and both parties work toward mutual benefits
the final decisions.
and collaborate on new product development
Remain as proprietary internal information and disclosed only on a need-to-know
basis.

Consider the different perspectives of project stakeholders, both outside the ‫التعليقات‬
organization and inside.
Contracts and a collaborative product development process should be in place and

.executed to confirm that a supplier development process has been implemented
‫اإلجابة الصحيحة‬

Consider the different perspectives of project stakeholders, both outside the


organization and inside.

https://docs.google.com/forms/d/e/1FAIpQLSd9z1GvRQYUImApP8VPctKUexYoQbfBNUSA1dJW6a8m_6nB7Q/viewscore?vc=0&c=0&w=1&flr=0&vie… 93/141 https://docs.google.com/forms/d/e/1FAIpQLSd9z1GvRQYUImApP8VPctKUexYoQbfBNUSA1dJW6a8m_6nB7Q/viewscore?vc=0&c=0&w=1&flr=0&vie… 94/141


‫ ص‬12:34 2023/‫‏‬5/‫‏‬13 ASQ CMQ-OE : Practice Test # 09 ‫ ص‬12:34 2023/‫‏‬5/‫‏‬13 ASQ CMQ-OE : Practice Test # 09

1/1* Which of the following assessment techniques provides the highest 1/1* As a key, global quality leader new to the organization, you have been
quality qualitative data? tasked with implementing a quality strategy for the business. The
organization's core business is a pharmaceutical supplier of powdered
goods to customers across multiple countries in Europe, Asia Pacific, and
Asking open-ended questions in an online or written mail survey
the Americas. Past Quality Management System (QMS) performance has
Using observations to record details on how customers use company products and caused a loss of confidence for key customers.
services
Top management wants an updated and relevant QMS to recover
Capturing customer comments or suggestions on the company website customer confidence and enable on-time delivery of quality goods
meeting relevant country and product safety protocols. You are required
Asking open-end questions in person so an interviewer can ask customers to
clarify their comments to:

1. Develop, deploy, and manage a QMS strategy across the global


organization.
‫التعليقات‬
2. Design and implement an effective measurement system that will
When in-person interviews are used, an interviewer can assess the ambiguity of a reflect the performance of the QMS and its strategy.

response to an open- ended question. The interviewer can then ask the customer to

.provide additional details or clarify what they mean
Effective KPI management for the QMS results in an identified gap. There
are multiple technologies that can be used to support an upgraded QMS.
The global team needs a system that stores its data and shares it across
the global business. Which of the following best reflects the ability to
store data that can be accessed

throughout the internet?

Autonomation

Automation

Machine Learning

Cloud Computing

‫التعليقات‬

Cloud computing is the practice of storing regularly used computer data on multiple
servers that can be accessed through the internet. In a global company with similar

.processes, this would be a benefit for KPI and data management

https://docs.google.com/forms/d/e/1FAIpQLSd9z1GvRQYUImApP8VPctKUexYoQbfBNUSA1dJW6a8m_6nB7Q/viewscore?vc=0&c=0&w=1&flr=0&vie… 95/141 https://docs.google.com/forms/d/e/1FAIpQLSd9z1GvRQYUImApP8VPctKUexYoQbfBNUSA1dJW6a8m_6nB7Q/viewscore?vc=0&c=0&w=1&flr=0&vie… 96/141


‫ ص‬12:34 2023/‫‏‬5/‫‏‬13 ASQ CMQ-OE : Practice Test # 09 ‫ ص‬12:34 2023/‫‏‬5/‫‏‬13 ASQ CMQ-OE : Practice Test # 09

1/1* Which of the following quality standards contain fundamentals and


vocabulary information?
0/1* To meet increased competition and demand, a U.S. producer has
outsourced several products to three suppliers in China. To ensure
success, which of the following communication techniques should be ISO 9001
used?
ISO 9000

A team comprised of purchasing, quality, and engineering personnel should meet ISO 9004
face-to-face with the suppliers and discuss timelines
ISO 19011
Certificate of Analysis for each product

Ongoing reports and periodic meetings to review contract requirements and


product performance to requirements ‫التعليقات‬

Periodic conference calls should be held to discuss product quality ISO 9000 covers the fundamental concepts of a quality system and definitions for

terminology used in the ISO 9001

.requirements) standard(
‫اإلجابة الصحيحة‬

Ongoing reports and periodic meetings to review contract requirements and product
performance to requirements

https://docs.google.com/forms/d/e/1FAIpQLSd9z1GvRQYUImApP8VPctKUexYoQbfBNUSA1dJW6a8m_6nB7Q/viewscore?vc=0&c=0&w=1&flr=0&vie… 97/141 https://docs.google.com/forms/d/e/1FAIpQLSd9z1GvRQYUImApP8VPctKUexYoQbfBNUSA1dJW6a8m_6nB7Q/viewscore?vc=0&c=0&w=1&flr=0&vie… 98/141


‫ ص‬12:34 2023/‫‏‬5/‫‏‬13 ASQ CMQ-OE : Practice Test # 09 ‫ ص‬12:34 2023/‫‏‬5/‫‏‬13 ASQ CMQ-OE : Practice Test # 09

0/1* As a key organizational leader with prior experience championing teams, 0/1* The human resources department is developing an anger management
you have been tasked with implementing a change management training program with a goal of focusing on the effects of typically
approach for the organization. The organization's core business is encountered real-life scenarios. Which type of training would the human
providing call center services to companies in the U.S. and Canada. Last resources team choose as the logical training design?
year, the organization began marketing and sales efforts to acquire
clients in Europe and China, and three new call centers have opened as
a result with more to follow.
Discussion format
Top management wants the change management approach to
Job aids
accomplish the following:
On-the-job training
1. Transition all call centers to an empowered, natural work team
structure. Experiential training format

2. Identify and develop change agents within the call center workforce ‫اإلجابة الصحيحة‬
that will drive continual improvement. 3. Establish buy-in on performance
metrics that will drive center efficiency and a high-degree of caller Experiential training format
satisfaction. You are given the resources to hire an external consultant as
a change agent for the initiative.

As part of an initial Force Field Analysis (FFA) and some follow-up


1/1* Which of the following situations is an example of the lean concept of
feedback from staff at all centers, it is determined that a team award
pull?
system must be used and not an individualized reward system. Why
would this approach be beneficial to both the organization and the center
staff? The upstream operation sets the pace for the downstream operation's production

The upstream operation produces to the schedule provided by the downstream


operation

Nothing is produced by the upstream operation until the downstream operation


Competition between staff and centers would be minimized
communicates it is ready
Individual performance reviews would be eliminated The upstream operation's production is synchronized to provide the downstream
operation just-in-time
Natural work teams will provide a means to establish ownership by center staff

The center manager's span of control would be reduced

‫التعليقات‬
‫اإلجابة الصحيحة‬
Upstream operations wait on the downstream signal that it needs what is being produced
Natural work teams will provide a means to establish ownership by center staff
.upstream. This is a pull system

https://docs.google.com/forms/d/e/1FAIpQLSd9z1GvRQYUImApP8VPctKUexYoQbfBNUSA1dJW6a8m_6nB7Q/viewscore?vc=0&c=0&w=1&flr=0&vie… 99/141 https://docs.google.com/forms/d/e/1FAIpQLSd9z1GvRQYUImApP8VPctKUexYoQbfBNUSA1dJW6a8m_6nB7Q/viewscore?vc=0&c=0&w=1&flr=0&vie… 100/141


‫ ص‬12:34 2023/‫‏‬5/‫‏‬13 ASQ CMQ-OE : Practice Test # 09 ‫ ص‬12:34 2023/‫‏‬5/‫‏‬13 ASQ CMQ-OE : Practice Test # 09

* Of the following options, which one best describes performance measures 1/1* Which of the following statements regarding Quality Function
that are critical for managing a project? Deployment (QFD) best represents its importance in the quality planning
process?

Balance sheet, income statement, earned value analysis, resource usage


QFD translates customer needs to product specifications and documents the
Resource usage, schedule timeliness, earned value analysis, budget variance correlation between them

Schedule timeliness, budget variance, balance sheet, earned value analysis QFD represents the voice of the customer, which is very important in quality
planning
Budget variance, resource usage, risk analysis, income statement
QFD matrices provide graphical representations that are easy to understand
‫اإلجابة الصحيحة‬
QFD involves the entire company in the design and control activity, addressing all
Resource usage, schedule timeliness, earned value analysis, budget variance stakeholder needs and concerns

‫التعليقات‬
0/1* Company ABC has trained supervisors on lean culture. The supervisor
QFD translates customer's needs into product specifications (design requirements) and
of a department is promoting this newly learned knowledge to newer
provides correlations between them. The house of quality helps us analyze and
employees. To support this initiative, what would the departmental understand data in the QFD matrix to provide the information needed to deliver product

.and service characteristics that customers really need and want
supervisor promote to the new employees as part of implementing the
lean culture?

Engage new employees on available "low hanging fruit" projects to give them
hands-on experience

Motivate the new employees to submit and lead the improvement projects

Engage new employees as team members in departmental projects aligned with


strategic planning

Motivate new employees to volunteer in other departmental improvement projects


to learn team building

‫اإلجابة الصحيحة‬

Engage new employees as team members in departmental projects aligned with


strategic planning

https://docs.google.com/forms/d/e/1FAIpQLSd9z1GvRQYUImApP8VPctKUexYoQbfBNUSA1dJW6a8m_6nB7Q/viewscore?vc=0&c=0&w=1&flr=0&vie… 101/141 https://docs.google.com/forms/d/e/1FAIpQLSd9z1GvRQYUImApP8VPctKUexYoQbfBNUSA1dJW6a8m_6nB7Q/viewscore?vc=0&c=0&w=1&flr=0&vie… 102/141


‫ ص‬12:34 2023/‫‏‬5/‫‏‬13 ASQ CMQ-OE : Practice Test # 09 ‫ ص‬12:34 2023/‫‏‬5/‫‏‬13 ASQ CMQ-OE : Practice Test # 09

1/1* The 10 plants of Company Z that all perform the same processes are
under one quality program with ISO 9001 as the guiding standard. 0/1* The Hoshin strategic planning model distinguishes itself from traditional
Process performance of each plant is measured using C. values. All strategic planning in that it:
plants have steady performance values of Cpk above 1.33, except for the
Laredo plant, which has Cpk variation between 0.5 and 1.1. It is puzzling
Begins with basic assumptions about the nature of the business, then tries to
to top management why this is happening given that the processes,
develop strategies to achieve objectives related to those assumptions.
procedures, and training programs are the same for all plants.
Is goal-/objective-oriented, vertically integrated, and has traceable hierarchy.
Which of the following would the Quality Director choose as a starting
point to evaluate the Laredo site so an improvement plan can be Incorporates operational and project planning, strategy formation and promotes
created? doing, reviewing, and planning.

Includes strategic assessment, strategic direction, strategic plans, program advice,


and a planning, programming, and budgeting system (PPBS).
Review procedure awareness and identify roadblocks to accessing them

‫اإلجابة الصحيحة‬
Review training records of the Laredo staff to ensure competency
Incorporates operational and project planning, strategy formation and promotes
Perform a survey to get input from process owners on aspects to fix doing, reviewing, and planning.

Perform a quality system audit of the Laredo plant

‫التعليقات‬

Since the 10 plants have similar or equivalent processes, procedures, and training, it is
likely the issues are systemic in nature and require a complete and thorough audit of the
quality system by unbiased auditors. The internal auditors from high-performing plants
could be used because they would easily identify system issues related to Laredo's
difference in performance. The Quality Director would authorize the audit and ensure that

.the report motivates Laredo to close gaps by adapting best practices from other plants

https://docs.google.com/forms/d/e/1FAIpQLSd9z1GvRQYUImApP8VPctKUexYoQbfBNUSA1dJW6a8m_6nB7Q/viewscore?vc=0&c=0&w=1&flr=0&vie… 103/141 https://docs.google.com/forms/d/e/1FAIpQLSd9z1GvRQYUImApP8VPctKUexYoQbfBNUSA1dJW6a8m_6nB7Q/viewscore?vc=0&c=0&w=1&flr=0&vie… 104/141


‫ ص‬12:34 2023/‫‏‬5/‫‏‬13 ASQ CMQ-OE : Practice Test # 09 ‫ ص‬12:34 2023/‫‏‬5/‫‏‬13 ASQ CMQ-OE : Practice Test # 09

1/1* Finding ways to avoid, reduce, or prevent the occurrence of risks and 0/1* When summarizing what has been discussed and decided during a
their associated losses are primarily the actions taken by which of the meeting, the team leader asks the person taking minutes to review their
following? notes. Why would this practice be important when concluding the
meeting?

Quality assurance process


This technique helps attendees correct the meeting minutes before the meeting
Quality control process ends

Quality auditing process This technique helps remind attendees of their agreements, thereby preventing
surprises later
Quality management review process
This technique allows the team leader to add in additional key points to the minutes
before the meeting ends

This technique provides the team with a final opportunity to verify or clarify their
‫التعليقات‬
understanding of what was discussed and decided before the meeting ends
Quality assurance involves the anticipation and identification of risks and potential issues
that require mitigation as a new project, product, or process is developed and ‫اإلجابة الصحيحة‬
implemented. Quality assurance prevents the occurrence of a problem; quality control

.prevents the recurrence of a problem
This technique provides the team with a final opportunity to verify or clarify their
understanding of what was discussed and decided before the meeting ends

0/1* A Six Sigma team completed their project, demonstrating an outstanding


team effort and a high degree of success. The VP of Quality recommends
that they apply for an award to recognize the team's performance. Which
of the following would be suitable for this recommendation?

International Team Excellence Award (ITEA)

Best Place to Work

Shingo Prize

Deming Prize

‫اإلجابة الصحيحة‬

International Team Excellence Award (ITEA)

https://docs.google.com/forms/d/e/1FAIpQLSd9z1GvRQYUImApP8VPctKUexYoQbfBNUSA1dJW6a8m_6nB7Q/viewscore?vc=0&c=0&w=1&flr=0&vie… 105/141 https://docs.google.com/forms/d/e/1FAIpQLSd9z1GvRQYUImApP8VPctKUexYoQbfBNUSA1dJW6a8m_6nB7Q/viewscore?vc=0&c=0&w=1&flr=0&vie… 106/141


‫ ص‬12:34 2023/‫‏‬5/‫‏‬13 ASQ CMQ-OE : Practice Test # 09 ‫ ص‬12:34 2023/‫‏‬5/‫‏‬13 ASQ CMQ-OE : Practice Test # 09

1/1* A company has just launched a product, but market reception is 1/1* You have just started as Quality Director at a manufacturing firm with
lukewarm because a competitor's product has 30% more power for the multiple locations, with 1500 employees total. You have found that
same size and price. Which of the following should the company procedural documentation is lacking and if there is any, it has not been
implement on the next product development project? updated in a long time. There are known issues with too much internal
rework and a lack of planning resources and efforts. Your short list has 25
possible areas to fix. The company COO has provided only a general
DFSS direction to increase capacity. Not all managers see the same issues, and
disagreement is common between departments and up and down the
PERT
management chain.
QFD
You have a large budget to work with as a large potential customer has
COQ
been identified, if you can meet their stringent cost and quality needs.
The company does not use the usual quality tools, and you were hired to
help.

‫التعليقات‬
You have decided to focus your efforts on your shipping area. In the
Shipping Department, procedures are unclear, and work tends to pile up
QFD determines how various product features meet customer needs and stack up against
and orders are often mixed up with each other. What tool is best to apply

.the competition so the best balance can be achieved
here?

Mistake proofing

Process mapping

Check sheets

Retraining

‫التعليقات‬

This area needs better work standardization. The various processes that are used need to
be agreed upon, and that means process mapping is needed to clear up confusion. As
process maps are made, the best approaches can be found and agreed upon, training can
be done, mistake proofing can be applied, and then check sheets used to track how often
.issues come up, defects are found, or functions performed

https://docs.google.com/forms/d/e/1FAIpQLSd9z1GvRQYUImApP8VPctKUexYoQbfBNUSA1dJW6a8m_6nB7Q/viewscore?vc=0&c=0&w=1&flr=0&vie… 107/141 https://docs.google.com/forms/d/e/1FAIpQLSd9z1GvRQYUImApP8VPctKUexYoQbfBNUSA1dJW6a8m_6nB7Q/viewscore?vc=0&c=0&w=1&flr=0&vie… 108/141


‫ ص‬12:34 2023/‫‏‬5/‫‏‬13 ASQ CMQ-OE : Practice Test # 09 ‫ ص‬12:34 2023/‫‏‬5/‫‏‬13 ASQ CMQ-OE : Practice Test # 09

1/1* In conducting a training needs assessment for frontline customer service 1/1* Which of the following would be more effective for a quality leader to sell
representatives, which of the following should be included? a new quality initiative to management?

Quantification of the results of performance problems Provide the big picture of the quality management approach to senior management
and share how this initiative fits with the big picture
Learning preferences of customer service representatives
Set up meetings with each department manager to present the proposal
Expected effectiveness of the training
Identify the risks and benefits involved in the initiative and seek a champion
Definition of the skill/knowledge gap to be closed
Solicit advice from a random sample of key organizational leaders to remove
obstacles that they mention

‫التعليقات‬

Defining the issue or skill gap that needs to be addressed is the traditional starting point in ‫التعليقات‬

.training needs assessment
Transforming risks and benefits into the language of management will likely secure
additional buy-in, especially if a champion backs the initiative and helps sell it to others on

.the management team

https://docs.google.com/forms/d/e/1FAIpQLSd9z1GvRQYUImApP8VPctKUexYoQbfBNUSA1dJW6a8m_6nB7Q/viewscore?vc=0&c=0&w=1&flr=0&vie… 109/141 https://docs.google.com/forms/d/e/1FAIpQLSd9z1GvRQYUImApP8VPctKUexYoQbfBNUSA1dJW6a8m_6nB7Q/viewscore?vc=0&c=0&w=1&flr=0&vie… 110/141


‫ ص‬12:34 2023/‫‏‬5/‫‏‬13 ASQ CMQ-OE : Practice Test # 09 ‫ ص‬12:34 2023/‫‏‬5/‫‏‬13 ASQ CMQ-OE : Practice Test # 09

1/1* A company wants to improve the way it handles customer complaints. 1/1* You are responsible for the full implementation of a new computer
Management is aware of the negative perceptions and experiences of system. What would be a prime consideration for designing the training?
customers and employees alike when dealing with the employees that
handle customer complaints at the call center. To create a positive
Demonstrating the advantages of the new system compared with the old one
change, the company needs to:
Taking adult learning principles into consideration

Ignore the customer complaints and reward call center employees.


Developing a schedule that works for instructors and participants

Align employee empowerment and involvement with customer needs.


Using computer-based training

Have a board of director's meeting to put new policies and procedures in place.

Provide negative reinforcement to the employees at the call center.


‫التعليقات‬

Adult learners always want to know WIIFM, or "What's in it for me?" By comparing the
advantages of the new system against the old system, you gain engagement, which in turn
‫التعليقات‬
.should drive knowledge transfer

Ignoring the needs of internal customers makes it very difficult to instill a desire to care for
the needs of external customers. A key method to energize internal customers to improve
products, processes, and services is to involve employees in designing and implementing

.strategies, procedures, and tools that facilitate customer satisfaction

1/1* You were recently certified as a team leader and have been assigned
your first project. How would you start?

Do a Force Field Analysis (FFA) and remove barriers that may stand in the way of
progress

Decide who you want on the team

Review the project charter with the sponsor and champion

Provide training on teamwork and quality tools to all potential team members

‫التعليقات‬

This is an appropriate first action so you better understand the project and can possibly

.suggest charter clarifications

https://docs.google.com/forms/d/e/1FAIpQLSd9z1GvRQYUImApP8VPctKUexYoQbfBNUSA1dJW6a8m_6nB7Q/viewscore?vc=0&c=0&w=1&flr=0&vie… 111/141 https://docs.google.com/forms/d/e/1FAIpQLSd9z1GvRQYUImApP8VPctKUexYoQbfBNUSA1dJW6a8m_6nB7Q/viewscore?vc=0&c=0&w=1&flr=0&vie… 112/141


‫ ص‬12:34 2023/‫‏‬5/‫‏‬13 ASQ CMQ-OE : Practice Test # 09 ‫ ص‬12:34 2023/‫‏‬5/‫‏‬13 ASQ CMQ-OE : Practice Test # 09

1/1* A process has been in place for several months and consistently 1/1* You are the Director of Quality and have recently worked with a third-
performing well within the upper and lower process limits. During the party auditing group to pursue ISO registration. Due to the prevailing
monthly management review, an instance was noted where one data culture in your organization and the hard work you invested in designing
point was well beyond the upper process limit. Which of the following the quality management system, the organization did well on the audit.
should be the response from management? One surprising non-conformance that was issued concerned how
documents within the quality manual were reviewed and approved. Since
you lead the Quality Department-and because your supervisor (the COO)
delegated the quality manual and the quality management system to you-
Immediately stop the process until the process owner can assure management that
no further instances or events outside of the process limits will occur you signed off on the revisions you introduced. As a result, you now need
to revise the process and the related procedure. That said, you are also
Gather data and identify causes serving as the interim Director of Training, so you need to ensure that the
members of the Quality Department know about and implement the
Put an immediate remedy in place to contain any damage
changes.
Determine the severity of the impact of this single point outside of the process
specifications Focusing on the training and development need as expressed in the
scenario, what is your first task?

Call a meeting of the Quality Department, but first design a training presentation in
‫التعليقات‬
PowerPoint
Since there is only one instance and no trends or indications of a process failure,
management should determine the impact in relation to severity of failure resulting from Option 2

.this single event
Define the need

Call a meeting of the Quality Department and begin training immediately

‫التعليقات‬

Avoid jumping into action without first defining the actual need of the training. Once the

.need is defined, you can move into design, delivery, and evaluation

https://docs.google.com/forms/d/e/1FAIpQLSd9z1GvRQYUImApP8VPctKUexYoQbfBNUSA1dJW6a8m_6nB7Q/viewscore?vc=0&c=0&w=1&flr=0&vie… 113/141 https://docs.google.com/forms/d/e/1FAIpQLSd9z1GvRQYUImApP8VPctKUexYoQbfBNUSA1dJW6a8m_6nB7Q/viewscore?vc=0&c=0&w=1&flr=0&vie… 114/141


‫ ص‬12:34 2023/‫‏‬5/‫‏‬13 ASQ CMQ-OE : Practice Test # 09 ‫ ص‬12:34 2023/‫‏‬5/‫‏‬13 ASQ CMQ-OE : Practice Test # 09

1/1 * To be effective with staff oversight, a Quality Manager would: 1/1* Relationships between departments are improved within an organization
so:

Attend training to learn job-specific tasks and communicate information regularly.


Teams are adequately staffed to improve processes.
Integrate staff and processes and take credit for work performed.
The business can accurately estimate order fulfillment dates and maintain on-time
Verify each task performed by staff and evaluate performance metrics against delivery.
goals.
The business understands gaps in operating the quality system.
Develop strategic plans to achieve corporate goals and provide direction to
staff to achieve specific results.
Each department understands and manages its suppliers and customers.

‫التعليقات‬
‫التعليقات‬
Giving direction to employees and developing strategic plans are both roles of an effective
.manager. These roles positively contribute to the manager's effectiveness This ensures that when one department makes a change to a process, they can make the

.change without hindering or surprising their upstream and downstream process owners

1/1* Entry of new, competitive substitute products, such as plastic in place of


metal in cars, most likely has the effect of:

Raising user purchase prices.

Increasing demand for metal products.

Lowering metal-producer selling prices.

Increasing chances that the Taft-Hartley Act will be invoked.

‫التعليقات‬

This is how Michael Porter's five forces play. A substitute most likely reduces the demand

.for the product being substituted

https://docs.google.com/forms/d/e/1FAIpQLSd9z1GvRQYUImApP8VPctKUexYoQbfBNUSA1dJW6a8m_6nB7Q/viewscore?vc=0&c=0&w=1&flr=0&vie… 115/141 https://docs.google.com/forms/d/e/1FAIpQLSd9z1GvRQYUImApP8VPctKUexYoQbfBNUSA1dJW6a8m_6nB7Q/viewscore?vc=0&c=0&w=1&flr=0&vie… 116/141


‫ ص‬12:34 2023/‫‏‬5/‫‏‬13 ASQ CMQ-OE : Practice Test # 09 ‫ ص‬12:34 2023/‫‏‬5/‫‏‬13 ASQ CMQ-OE : Practice Test # 09

1/1* Of the following actions a customer can take to communicate with 1/1* Common obstacles to the team-building process occur when managers
suppliers, which would be considered proactive? expect results too quickly and push the team to work on tasks
immediately. Which of the following would be premature for management
to expect early in the team growth process?
Send a request to the supplier to rework their inventory

Identify control plan items that the supplier impacts


Conduct a meeting

Recommend changes to a corrective action report for defective product


Resolve the charter

Notify the supplier of delinquent corrective action reports


Establish team ground rules

Demonstrate effective brainstorming

‫التعليقات‬


.Control plans are written descriptions of the systems used to minimize product variation
‫التعليقات‬

The team will need to use brainstorming later as they work on aspects of the problem and
solutions. Note that this skill evolves over time. During the initiate and forming stages, the
team needs to conduct good, effective meetings and decide on ground rules and charter
content. These things get done with effective decision making and do not require much

.brainstorming

https://docs.google.com/forms/d/e/1FAIpQLSd9z1GvRQYUImApP8VPctKUexYoQbfBNUSA1dJW6a8m_6nB7Q/viewscore?vc=0&c=0&w=1&flr=0&vie… 117/141 https://docs.google.com/forms/d/e/1FAIpQLSd9z1GvRQYUImApP8VPctKUexYoQbfBNUSA1dJW6a8m_6nB7Q/viewscore?vc=0&c=0&w=1&flr=0&vie… 118/141


‫ ص‬12:34 2023/‫‏‬5/‫‏‬13 ASQ CMQ-OE : Practice Test # 09 ‫ ص‬12:34 2023/‫‏‬5/‫‏‬13 ASQ CMQ-OE : Practice Test # 09

1/1* A pharmaceutical sales representative needs to place an order for 1/1* Carl's Car Company has been tracking the cost of quality for several
medications for a surgery center. To get the order processed, who will the years. During this time, internal failure costs have risen slightly by $3,000
sales representative need to contact? per month, but the overall cost of quality has decreased from 10% to 5%
of sales. The budget and amount spent on prevention and appraisal has
remained constant over this time. Which of the following would be an
Pharmaceutical manufacturing quality control inspector
appropriate course for future action?
Surgery center CEO

Analyze internal and external failure cost breakdown


Pharmaceutical company customer service representative

Increase budget for training and inspection activities


Surgery center customer service representative

Review the program to ensure that all quality costs are captured

Continue current programs and identify improvement opportunities


‫التعليقات‬

The internal Customer Service Department handles orders for customers. This is an
example of the internal customer with the ultimate responsibility and purpose to serve the

.external customer
‫التعليقات‬

Since total quality costs are decreasing as percentage of sales, current programs are

.working and should be continued

https://docs.google.com/forms/d/e/1FAIpQLSd9z1GvRQYUImApP8VPctKUexYoQbfBNUSA1dJW6a8m_6nB7Q/viewscore?vc=0&c=0&w=1&flr=0&vie… 119/141 https://docs.google.com/forms/d/e/1FAIpQLSd9z1GvRQYUImApP8VPctKUexYoQbfBNUSA1dJW6a8m_6nB7Q/viewscore?vc=0&c=0&w=1&flr=0&vie… 120/141


‫ ص‬12:34 2023/‫‏‬5/‫‏‬13 ASQ CMQ-OE : Practice Test # 09 ‫ ص‬12:34 2023/‫‏‬5/‫‏‬13 ASQ CMQ-OE : Practice Test # 09

1/1* Given the results of the following human capital readiness report, on * A company is organized by function. An improvement team has concluded
which job family should more hiring efforts be focused? that the process changes they are working on involves several of these
functions. Who would be the best person to address this situation with
process stakeholders?

Team facilitator

Team sponsor

Team leader
Process quality engineers
Team member
Supplier quality auditors

Internal quality auditors

Program quality managers ‫التعليقات‬

A team sponsor is typically senior enough to secure the commitment of other leaders and

.peers that own processes in different functions

‫التعليقات‬

Program or project quality managers require a longer timeframe to get up to speed


because they are responsible for managing and controlling the quality of work over an
extended period. Given the number of open positions and current capacity in this job 0/1* You are the newly appointed training manager at a rapidly growing brick-

.family, this need demands primary hiring focus
and-mortar retailer that values the attitudes of new hires far more than
specific job knowledge. How would you start developing a customer
service training program for new hires?

Make effective use of available technologies, both for training and for service
delivery

Provide a detailed orientation on the company's history and its customer


service goals

Define the ideal customer service experience from both a customer and employee
perspective

Review what competitors are doing and design a program based on best practices

‫اإلجابة الصحيحة‬

Define the ideal customer service experience from both a customer and employee
perspective

https://docs.google.com/forms/d/e/1FAIpQLSd9z1GvRQYUImApP8VPctKUexYoQbfBNUSA1dJW6a8m_6nB7Q/viewscore?vc=0&c=0&w=1&flr=0&vie… 121/141 https://docs.google.com/forms/d/e/1FAIpQLSd9z1GvRQYUImApP8VPctKUexYoQbfBNUSA1dJW6a8m_6nB7Q/viewscore?vc=0&c=0&w=1&flr=0&vie… 122/141


‫ ص‬12:34 2023/‫‏‬5/‫‏‬13 ASQ CMQ-OE : Practice Test # 09 ‫ ص‬12:34 2023/‫‏‬5/‫‏‬13 ASQ CMQ-OE : Practice Test # 09

1/1* You are a process engineer at a leading manufacturer of custom-built 1/1* A review of the concepts and philosophies of the major quality gurus
components for customers in the aircraft industry. Although product would indicate which of the following to be the most effective way to
quality is not an issue, late delivery is now the top reason for customer ensure quality?
complaints. You have been asked to lead a team to see what should be
done. How should you proceed?
Benchmarking best competitive practices

Scientific problem solving

Create a value stream map to identify problems


Statistical process control

Communicate the problem to all the managers who have input


Continuous improvement

Conduct an analysis of late deliveries to see if there are any patterns

Establish goals and objectives for key processes


‫التعليقات‬

Continuous improvement is the approach that all historical quality gurus indicated as
essential for ensuring quality (i.e., kaizen). There are various specific practices associated
‫التعليقات‬ with this approach, including those mentioned in the other three answers. Quality comes
from a well- designed process that is continuously improved to minimize or eliminate the

.need for inspection
This should yield some insights very quickly and may point to the root cause. It will
establish what process is to be mapped (if necessary), what will be communicated to

.managers, and what established objectives are not being accomplished

https://docs.google.com/forms/d/e/1FAIpQLSd9z1GvRQYUImApP8VPctKUexYoQbfBNUSA1dJW6a8m_6nB7Q/viewscore?vc=0&c=0&w=1&flr=0&vie… 123/141 https://docs.google.com/forms/d/e/1FAIpQLSd9z1GvRQYUImApP8VPctKUexYoQbfBNUSA1dJW6a8m_6nB7Q/viewscore?vc=0&c=0&w=1&flr=0&vie… 124/141


‫ ص‬12:34 2023/‫‏‬5/‫‏‬13 ASQ CMQ-OE : Practice Test # 09 ‫ ص‬12:34 2023/‫‏‬5/‫‏‬13 ASQ CMQ-OE : Practice Test # 09

1/1* When planning for incoming levels of inspection of purchased materials, 1/1* A drug manufacturer with a history of poor financial performance, drug
the quality manager should consider which of the following aspects of the recalls, and FDA warnings declares bankruptcy and is purchased by
product? another company. The new company introduces cost-savings plans to
break past cycles of over-spending and adds a focus on marketing to
new customers. There is also a renewed effort to ensure that
The monetary value of the product
manufacturing quality management meets FDA standards. The methods
The lot size of product and the frequency of delivery for the product and tools to do this are implemented, but the new organization continues
to welcome and use input and help from employees to further improve
The final customer or end user of the product .production processes and new customer acquisition

If the product is made-to-stock or made-to-order What would this company's approach to change establish?

Change movement
‫التعليقات‬
Unfreezing
The frequency of delivery and lot size should be elements considered when developing an

.inspection plan
Refreezing

Change agents

‫التعليقات‬
* Quality Tests R Us recently advertised a new position for a Quality Manager.
Which applicant summary best exhibits the better leadership traits for the Given that changes are in place and people have accepted new ways of working, the
company is now in the refreeze stage of change. Changes are being used, they are
Quality Manager position?
.incorporated into everyday business, and ongoing continual improvement is occurring

Applicant D monitors team performance, takes appropriate corrective actions, and


consistently seeks continual improvement opportunities

Applicant B is a great communicator and skilled at selecting, hiring, and training


personnel

Applicant C directs employees on ways to be productive and acquires resources


needed to accomplish quality system tasks

Applicant A is goal-oriented, inherently understands people and systems, and


champions the success of teams

‫التعليقات‬

This applicant is exhibiting several leadership traits of a Quality Manager: a system


.perspective, empowerment and interpersonal relationships, and future state orientation

https://docs.google.com/forms/d/e/1FAIpQLSd9z1GvRQYUImApP8VPctKUexYoQbfBNUSA1dJW6a8m_6nB7Q/viewscore?vc=0&c=0&w=1&flr=0&vie… 125/141 https://docs.google.com/forms/d/e/1FAIpQLSd9z1GvRQYUImApP8VPctKUexYoQbfBNUSA1dJW6a8m_6nB7Q/viewscore?vc=0&c=0&w=1&flr=0&vie… 126/141


‫ ص‬12:34 2023/‫‏‬5/‫‏‬13 ASQ CMQ-OE : Practice Test # 09 ‫ ص‬12:34 2023/‫‏‬5/‫‏‬13 ASQ CMQ-OE : Practice Test # 09

1/1* In a hospital, lower-level staff are members of special-purpose teams. 1/1* As the Quality Manager, you have created supplier training material on
The leaders of these teams are members of higher-level teams whose how to clarify business needs and quality requirements. You copyrighted
leaders are members of the senior management team. This structure the material and shared it with your supply base. What risk is associated
enhances inter-team coordination and is commonly known as: with sharing this information?

Hoshin planning Quality

Interlocking structure Impact

Cross-functional hierarchy Ethical

Chain of command Intellectual property

‫التعليقات‬ ‫التعليقات‬

A structure like the one described is referred to as interlocking structure of teams. The Intellectual property is any product created by the mind or intellect that the law protects
structure is set up to align various teams on key organizational goals like culture change, from unauthorized use by others. One common type of intellectual property is a copyright.
ISO certification, strategic planning deployment, strategic product development, mergers,
.Your suppliers might use this information without your permission

.or new market pursuits

1/1* A customer is in process of closing a corrective action issued to a


supplier for defective material. Which of the following should the
customer choose as the next step to take?

Invite the supplier to attend in-house training provided by the customer

Send a customer technical representative to assist the supplier

Set up a meeting to discuss the issue

Monitor the next shipments

‫التعليقات‬

Monitoring the next shipments would be the next step as the customer needs to verify the
.effectiveness of the action plan taken

https://docs.google.com/forms/d/e/1FAIpQLSd9z1GvRQYUImApP8VPctKUexYoQbfBNUSA1dJW6a8m_6nB7Q/viewscore?vc=0&c=0&w=1&flr=0&vie… 127/141 https://docs.google.com/forms/d/e/1FAIpQLSd9z1GvRQYUImApP8VPctKUexYoQbfBNUSA1dJW6a8m_6nB7Q/viewscore?vc=0&c=0&w=1&flr=0&vie… 128/141


‫ ص‬12:34 2023/‫‏‬5/‫‏‬13 ASQ CMQ-OE : Practice Test # 09 ‫ ص‬12:34 2023/‫‏‬5/‫‏‬13 ASQ CMQ-OE : Practice Test # 09

0/1* Empowering employees is a necessity for sustaining quality and very 1/1* ISO 9001 requirements specifically address all but which of the following?
common in today's workplace. One of the methods often used in
empowering employees is job enrichment. How is job enrichment best
Document control
?described
Social responsibility

Fitting the job to the worker so that the highest level of productivity is achieved Risk management

Establish the organizational requirements as well as the position needs and Technology, buildings and utilities
requirements of job holders

Provide tasks and responsibilities to make the job holder more skilled and
competent to perform expected work
‫التعليقات‬
Build a higher sense of challenge and achievement into the job so they lead to Social responsibility includes many characteristics like community support, waste
personal and career growth management, environmental protection, and legal/regulatory compliance. ISO 9001 covers
some, but not all, of these characteristics. The Baldrige criteria are more comprehensive in
‫اإلجابة الصحيحة‬
.terms of social responsibility and being a good corporate citizen

Build a higher sense of challenge and achievement into the job so they lead to
personal and career growth

https://docs.google.com/forms/d/e/1FAIpQLSd9z1GvRQYUImApP8VPctKUexYoQbfBNUSA1dJW6a8m_6nB7Q/viewscore?vc=0&c=0&w=1&flr=0&vie… 129/141 https://docs.google.com/forms/d/e/1FAIpQLSd9z1GvRQYUImApP8VPctKUexYoQbfBNUSA1dJW6a8m_6nB7Q/viewscore?vc=0&c=0&w=1&flr=0&vie… 130/141


‫ ص‬12:34 2023/‫‏‬5/‫‏‬13 ASQ CMQ-OE : Practice Test # 09 ‫ ص‬12:34 2023/‫‏‬5/‫‏‬13 ASQ CMQ-OE : Practice Test # 09

1/1*  Which of the following training planning factors would come last in order 1/1* One of your tasks is to do a preliminary evaluation of capital project
sequence? proposals. In reviewing submissions, you find that the proposal with the
fastest payback does not appear to support any of the three major
elements of the corporate strategy.
Determine the needs
What would you do?
Define the objectives

Determine the course content and structure Review the proposal with the project proposer and invite further discussion of
the strategic support the project offers
Prepare the necessary material
Challenge the assumptions which lead to the rapid payback

Reject the proposal and outline your reasons to the project proposer
‫التعليقات‬
Do not include this proposal in your list of recommendations to management
Actually the answer choices are already listed in appropriate sequence (A, B, C, D). Another
way to approach the question is that the need for training, the training objectives and the
.content and structure help shape the training materials

‫التعليقات‬

This may be an excellent project to pursue, but it has not been positioned correctly. A
basic characteristic of a tactical plan is that it is linked to strategic objectives. More dialog

.with the proposer may help with strategic alignment of the proposal

https://docs.google.com/forms/d/e/1FAIpQLSd9z1GvRQYUImApP8VPctKUexYoQbfBNUSA1dJW6a8m_6nB7Q/viewscore?vc=0&c=0&w=1&flr=0&vie… 131/141 https://docs.google.com/forms/d/e/1FAIpQLSd9z1GvRQYUImApP8VPctKUexYoQbfBNUSA1dJW6a8m_6nB7Q/viewscore?vc=0&c=0&w=1&flr=0&vie… 132/141


‫ ص‬12:34 2023/‫‏‬5/‫‏‬13 ASQ CMQ-OE : Practice Test # 09 ‫ ص‬12:34 2023/‫‏‬5/‫‏‬13 ASQ CMQ-OE : Practice Test # 09

1/1* A work team has identified three software packages that will help 2/2* Every year, the executive management team issues updated strategy
dramatically improve a process and needs to decide which to purchase. planning based on changes in the internal and external variables. Each
Each package has some similar and some different features. There are department is required to submit departmental objectives for the year.
many different criteria that need to be used in making the decision. Which
of the following tools is likely to be most valuable for helping the team What action should be taken by the executive management team to
decide? ensure effectiveness of the program?

Matrix diagram
Establish measurement criteria for objectives and ensure relevance to the
Activity Network Diagram (AND) overall goal

Allow the department managers to monitor progress


Priorities matrix

Accept all improvement initiatives contributed by the department as value-added


Tree diagram

Identify trends and uncertainties leading into the future

‫التعليقات‬

The priorities matrix assists in choosing between several options that have many useful ‫التعليقات‬
.benefits, but where not all of them are of equal value
An effective method of setting objectives is establishing measurements to obtain

.benchmarks and measurement of success

https://docs.google.com/forms/d/e/1FAIpQLSd9z1GvRQYUImApP8VPctKUexYoQbfBNUSA1dJW6a8m_6nB7Q/viewscore?vc=0&c=0&w=1&flr=0&vie… 133/141 https://docs.google.com/forms/d/e/1FAIpQLSd9z1GvRQYUImApP8VPctKUexYoQbfBNUSA1dJW6a8m_6nB7Q/viewscore?vc=0&c=0&w=1&flr=0&vie… 134/141


‫ ص‬12:34 2023/‫‏‬5/‫‏‬13 ASQ CMQ-OE : Practice Test # 09 ‫ ص‬12:34 2023/‫‏‬5/‫‏‬13 ASQ CMQ-OE : Practice Test # 09

1/1* After getting no significant tangible results from work done by two outside 1/1* The quality manager designs a customer survey while implementing a
consulting firms, a company is considering engaging your firm to get the customer feedback and improvement process. What is the quality
results they have been unable to achieve so far. What action would you manager hoping to achieve?
suggest to them as a first step toward achieving their objective?

A reactive vs. proactive approach


Establish or revise the quality policy and vision for the entire company
A process with a large capital investment to modify
Benchmark their current results against that of similar companies
Determination of the critical path method
Perform an assessment of past failures and successes in achieving their
desired results
A Plan-Do-Check-Act (PDCA) process within a PDCA process
Use the Baldrige criteria for performance excellence as a framework for conducting
an organizational gap analysis

‫التعليقات‬

A PDCA process is being used within a larger PDCA process to determine the
‫التعليقات‬ effectiveness of the process improvement activity for customer satisfaction. The action is

.initiated by developing a plan for improvement, followed by putting the plan into action
Unless you have a clear understanding of what has been tried previously, you run the risk
of repeating the same mistakes and creating even more cynicism within the company.
Results may also need to be clarified and translated into a set of SMART goals linked to

.strategy to get traction on ways to achieve desired results

https://docs.google.com/forms/d/e/1FAIpQLSd9z1GvRQYUImApP8VPctKUexYoQbfBNUSA1dJW6a8m_6nB7Q/viewscore?vc=0&c=0&w=1&flr=0&vie… 135/141 https://docs.google.com/forms/d/e/1FAIpQLSd9z1GvRQYUImApP8VPctKUexYoQbfBNUSA1dJW6a8m_6nB7Q/viewscore?vc=0&c=0&w=1&flr=0&vie… 136/141


‫ ص‬12:34 2023/‫‏‬5/‫‏‬13 ASQ CMQ-OE : Practice Test # 09 ‫ ص‬12:34 2023/‫‏‬5/‫‏‬13 ASQ CMQ-OE : Practice Test # 09

1/1* Your company, a major pharmaceutical manufacturer, is going to 1/1* Effective actions taken to improve products and services provided by a
introduce automation to several lines that have been successfully using sports clothing manufacturer to the distributors of their athletic products
lean principles for some time. Included in this project is computer based could include:
SPC. What is your starting point for assessing the training needs of the
affected operators?
Identifying patterns of sales by product, geographic location, and size of
distributor.
Develop self-directed learning modules so that the operators can learn new
Identifying the competition to determine if other manufacturers are using the same
knowledge and skills at their own pace
distributors.
Train operators on regulatory standards for this kind of process Identifying the salespersons who buy the product from the distributors and sell
your product.
Determine each operator's familiarity with using computers and SPC principles
to control a process Identifying the credibility of the provider, range of services offered, and degree of
customization offered.
Assess whether management is committed to the training investment

‫التعليقات‬
‫التعليقات‬
When the manufacturer seeks a better understanding of customer requirements by
Depending on familiarity, you may first need to train operators on the basics of using the segment (products purchased, sales volume, geographic location, etc.), they will be more

.technology and/ or SPC before they can correctly apply training resources effective in providing the right products and services to meet key customer requirements

.for each segment

1/1* The use of QCI's CMQ/OE study materials to pass ASQ's certification
exam is most typical of what training effect measurement level?

Learning

Behavior

Results

https://docs.google.com/forms/d/e/1FAIpQLSd9z1GvRQYUImApP8VPctKUexYoQbfBNUSA1dJW6a8m_6nB7Q/viewscore?vc=0&c=0&w=1&flr=0&vie… 137/141 https://docs.google.com/forms/d/e/1FAIpQLSd9z1GvRQYUImApP8VPctKUexYoQbfBNUSA1dJW6a8m_6nB7Q/viewscore?vc=0&c=0&w=1&flr=0&vie… 138/141


‫ ص‬12:34 2023/‫‏‬5/‫‏‬13 ASQ CMQ-OE : Practice Test # 09 ‫ ص‬12:34 2023/‫‏‬5/‫‏‬13 ASQ CMQ-OE : Practice Test # 09

0/1* Project Delta has been problematic since its kick-off. After 6 months of 1/1* A company's customer complaint processing program, deployed 2 years
mitigating operational issues, enough data has been gathered for a ago, seems to be losing effectiveness. Root cause analysis into the loss
financial report. Contrary to the positive productivity reports issued by of effectiveness, at times, is superficial due to process owner time
operations, the financial analysis showed a trend of lost margin. A lean- constraints with other daily priorities. Recurrence of complaint issues is
based steering team was assigned to evaluate this situation to address increasing. The Quality Director plans to evaluate the quality plan to
the immediate issue with this account and generate lessons learned for mitigate or, if needed, change some plan elements to reverse this trend.
future planning/account management.
Which of the following should the Quality Director evaluate to mitigate or
Which of the following should the steering committee take as a first step correct the trend?
in analyzing the situation?

Evaluate if customer complaint Pareto reporting is kept up-to-date and accessible


Define the problem and the organizational requirement to process owners

Evaluate if customer requirements for all support and core processes are
Improve the measurement variables or metrics at all phases of operations
understood, captured, and implemented in process design and control

Identify the root causes that have caused the problem Evaluate the quality system's internal audit program to determine if heightened
frequency should be implemented for problematic areas like corrective action,
Develop a change management program to execute improvement plans staff competency, and/or others as identified by the Director

‫اإلجابة الصحيحة‬ Evaluate if metrics for each core process are appropriate to provide actionable
feedback on customer satisfaction
Define the problem and the organizational requirement

‫التعليقات‬

If customer requirements are not understood and deployed at all phases of the operation,
discrepancies/complaints occur. The first step is to check if this element is effectively
deployed and then strengthen the aspects of the quality plan to build and sustain it over

.time

https://docs.google.com/forms/d/e/1FAIpQLSd9z1GvRQYUImApP8VPctKUexYoQbfBNUSA1dJW6a8m_6nB7Q/viewscore?vc=0&c=0&w=1&flr=0&vie… 139/141 https://docs.google.com/forms/d/e/1FAIpQLSd9z1GvRQYUImApP8VPctKUexYoQbfBNUSA1dJW6a8m_6nB7Q/viewscore?vc=0&c=0&w=1&flr=0&vie… 140/141


‫ ص‬12:34 2023/‫‏‬5/‫‏‬13 ASQ CMQ-OE : Practice Test # 09
‫ سياسة الخصوصية‬- ‫ شروط الخدمة‬- .Google ‫لم يتم إنشاء هذا المحتوى وال اعتماده من ِقبل‬

 ‫نماذج‬

https://docs.google.com/forms/d/e/1FAIpQLSd9z1GvRQYUImApP8VPctKUexYoQbfBNUSA1dJW6a8m_6nB7Q/viewscore?vc=0&c=0&w=1&flr=0&vie… 141/141

You might also like